adasap.pdf

68
American Diabetes Association Self-Assessment Program Multiple Choice Questions Assessment and Educational (Learning) Critiques Components ADA-SAP Module 1 Basic Principles of Management of Type 2 Diabetes Cure Care Commitment ® American Diabetes Association® ® © 2007 American Diabetes Association. All rights reserved.

Upload: krairat-komdee

Post on 21-Dec-2015

13 views

Category:

Documents


3 download

DESCRIPTION

Self Assessment

TRANSCRIPT

American Diabetes AssociationSelf-Assessment Program

Multiple Choice Questions Assessment and Educational (Learning) Critiques Components

ADA-SAP™

K Module 1Basic Principles of Management of Type 2 Diabetes

C u r e C a r e C o m m i t m e n t ®

AmericanDiabetesAssociation®®

© 2007 American Diabetes Association. All rights reserved.

PEIDeveloped and Published by: Professional Evaluation, Inc.Developing Medical Specialty Board Category 1 CME Programs for Over Three Decades

Professional Evaluation Inc.4 Midland Avenue, Suite 105, Berwyn, PA 19312-1687 | www.proevalinc.com

Table of Contents

3 Learning Objectives

3 Accreditation

4 Faculty

4 Disclosure Statement

5 General Instructions

6 Multiple Choice Questions (MCQs)

25 Appendix

26 I. Diagnosis of impaired glucose tolerance (IGT) and impaired fasting glucose (IFG)

27 II. Diagnosis of type 2 diabetes

28 III. Summary of recommendations for adults with diabetes

34 Educational Critiques

65 Computer-scored Answer Sheet

66 Program Evaluation Questions

American Diabetes AssociationSelf-Assessment Program (ADA-SAPTM)

K Module 1Basic Principles of Management of Type 2 Diabetes

American Diabetes Association Self-Assessment Program2

Dear Health Care Professional:

The American Diabetes Association (ADA), in cooperation with Professional Evaluation, Inc. (PEI),has developed the American Diabetes Association Self-Assessment Program (ADA-SAP TM) series.Module 1 of ADA-SAPTM series covers the Basic Principles of Diagnosis and Management of Type 2 Diabetes in adults. Future modules in the ADA-SAPTM series will cover topics such as, combination therapy for type 2 diabetes, insulin use, and management of co-morbidities. For additional information,visit PEI’s website, www.proevalinc.com.

The multiple choice questions component of Module 1 of ADA-SAPTM was developed by PEI at the upper level of difficulty similar in type and format to a board certifying examination. It was designed to objectively assess, strengthen and reinforce your knowledge and to provide you with an in-depth learning experience of active engagement. Once you have properly completed Module 1 in it’s entirety (see General Instructions on page 5), your knowledge of the approach to diagnosis and management of type 2 diabetes in adults should be greatly enhanced.

The ADA looks forward to your participation in this important new program of continuing education,as part of your commitment to lifelong learning.

Sincerely,

Michael H. Davidson M.D., FACC, FACPProgram ChairChief Medical Officer, PEIClinical Professor of Medicine, Director of Preventive CardiologyThe University of Chicago Pritzker School of Medicine

C u r e C a r e C o m m i t m e n t ®

AmericanDiabetesAssociation®®

3American Diabetes Association Self-Assessment Program

Target AudiencePrimary Care Physicians, General Internists, FamilyPhysicians, Doctors of Osteopathy, Registered Nurses,Nurse Practitioners, Pharmacists, Dietitians, andCertified Diabetes Educators.

Learning ObjectivesBy completing this CME/CE enduring materials activity (Module 1), participants will be better able to:K Discuss the pathogenesis of insulin resistance

and pancreatic β-cell dysfunction leading to type 2diabetes.

K Interpret and utilize ADA Guidelines and treatment algorithms.

K Implement non-pharmacological and pharma-cological treatment strategies for hyperglycemia in type 2 diabetes.

This enduring materials activity was originally released on November 15, 2007 and terminated on November 30, 2009.

FacultyK ChairMichael H. Davidson, M.D., FACC, FACPClinical ProfessorDirector of Preventive Cardiology The University of ChicagoExecutive Medical DirectorRadiant ResearchChicago, IL

K FacultyElliot B. Davidson, M.D., FAAFPFamily MedicineAkron General Medical CenterAkron, OH

Stanley Schwartz, M.D., FACPClinical Associate Professor of MedicineDirector, Diabetes Disease ManagementUniversity of Pennsylvania Health SystemPhiladelphia, PA

K Chief Editor and PublisherDante S. LaRocca, Ph.D.PresidentProfessional Evaluation, Inc.Ambler, PA

K ReviewersM. Sue Kirkman, M.D.Vice President, Clinical AffairsAmerican Diabetes AssociationAlexandria, VA

Belinda P. Childs, ARNP, MN, CDE, BC-ADMClinic and Research CoordinatorMid-America Diabetes Associates, PAWichita, KS

Russell D. White, M.D., FAFP, FACSM, MACEProfessor of MedicineDirector,Sports Medicine Fellowship ProgramUniversity of MissouriKansas City School of MedicineTruman Medical Center—LakewoodKansas City, MO

This activity was supported by an unrestricted educational grant from Merck & Co., Inc.

Disclosure Statement All participating faculty, course directors, and planning committee members are required to disclose to the program audienceany financial relationships related to the subject matter of this program. Disclosure information is reviewed in advance in order to manage and resolve any possible conflicts of interest. The intent of this disclosure is not to prevent a planner orpresenter from being involved in the activity, but rather to provide participants with information on which they can make their own judgments.

Content Validation Statement The American Diabetes Association’s accredited continuing education activities are based on evidence that is acceptedwithin the profession of medicine. Scientific research referred to, reported, or used in support or justification of a patient care recommendation conforms to the generally accepted standards of experimental design, data collection and analysis.

Unapproved Uses of Drugs/Devices In accordance with requirements of the FDA, the audience is advised that information presented in this continuing education activity may contain references to unlabeled or unapproved uses of drugs or devices. Please refer to the FDAapproved package insert for each drug/device for full prescribing/utilization information.

Belinda P. Childs, ARNP, MN, CDE, BC-ADMResearch Support: Amylin, Bayer Healthcare, Biodel, Eli Lilly, Novartis, Roche Diagnostic, sanofi-aventis; Speakers Bureau: Amylin, Eli Lilly; Advisory Panel: Amylin; Consultant: Roche Diagnostic, sanofi-aventis.

Elliot B. Davidson, M.D., FAAFPSpeakers Bureau: AstraZeneca, Novartis, Reliant Pharmaceuticals, Inc.

Michael H. Davidson, M.D., FACCResearch Support: Abbott Laboratories, AstraZeneca Pharmaceuticals, Daiichi-Sankyo, Inc., Merck/Schering-Plough,Pfizer Laboratories, Reliant Pharmaceuticals, Inc., Roche Pharmaceuticals, sanofi-aventis, Takeda Pharmaceuticals;Speakers Bureau: Abbott Laboratories, AstraZeneca Pharmaceuticals, Daiichi-Sankyo Inc., Merck & Co. Inc., MerckSchering Plough, Pfizer Inc., Reliant Pharmaceuticals, Inc., Takeda Pharmaceuticals; Consultant: Abbott Laboratories,AstraZeneca Pharmaceuticals, Daiichi-Sankyo Inc., Merck & Co. Inc., Merck Schering-Plough, Pfizer Inc., ReliantPharmaceuticals, Inc., Takeda Pharmaceuticals.

M. Sue Kirkman, M.D.Disclosed no conflict of interest.

Dante S. LaRocca, Ph.D.Disclosed no conflict of interest.

Stanley Schwartz, M.D., FACPSpeakers Bureau: Amylin, Eli Lilly, Merck & Co. Inc., Novo Nordisk, Inc., Pfizer Inc., sanofi-aventis, TakedaPharmaceuticals; Advisory Panel: Eli Lilly, Takeda Pharmaceuticals.

Russell D. White, M.D.Speaker Bureau: American Academy of Family Physicians, The National Procedures Association, University of Missouri-Kansas City; Consultant: Novo Nordisk, Inc.

American Diabetes Association Self-Assessment Program4

5American Diabetes Association Self-Assessment Program

General Instructions for Completion of ADA-SAP™

Module 1 of ADA-SAP™ consists of the following components:

K Multiple Choice Questions (MCQs) Assessment ComponentThis consists of 75 clinical problem-solving MCQs. Your task is to select the one lettered option among four or five offered that BEST answers each MCQ.

K AppendixThis is a comprehensive summary of the recommendations of the ADA for diagnosis and management of type 2 diabetes. It is based on peer-reviewed publications and is designed to assist you to complete the Assessment and Educational Components of ADA-SAP™.

K Answer SheetAn answer sheet is provided for you to record your answers as you work through the MCQs component of the program. Refer to this answer sheet to assess your performance while reading the Critiques component.

K Educational (Learning) Critiques ComponentIN ORDER TO MAXIMIZE YOUR LEARNING EXPERIENCE IT IS IMPORTANT THAT YOU DONOT LOOK AT THE EDUCATIONAL CRITIQUES COMPONENT UNTIL AFTER YOU HAVECOMPLETED MARKING YOUR ANSWERS FOR THE MCQs ON THE ANSWER SHEET. Thecritiques component contains detailed explanations for the correct and incorrect answers for the MCQs basedon the most current peer-reviewed published information. Once you have read the Educational Critiques, thebibliographic references should be utilized as follow-up study for those MCQs which you answered incorrectly.The critiques are the teaching and learning component of ADA-SAP™. They are to be used in combination with the MCQ assessment component to provide you with a positive, active learning experience.

DO NOT CHANGE THE ANSWERS MARKED ON YOUR ANSWER SHEET WHILE READINGTHE CRITIQUES. THE INTENT OF ADA-SAP™ IS LEARNING NEW KNOWLEDGE AND REINFORCING PREVIOUSLY LEARNED KNOWLEDGE.

K Program FeedbackAfter you have completed the MCQ and Critique components , you may email any comments about the contentor format of this Self-Assessment Program to [email protected].

American Diabetes AssociationSelf-Assessment ProgramMultiple Choice Questions (MCQs)

American Diabetes Association Self-Assessment Program MCQs6

7MCQs American Diabetes Association Self-Assessment Program

Multiple Choice Questions (MCQs)Items 1–75

DirectionsItems 1–75 consist of a question or an incomplete statement followed by three, four or five lettered options.Your task is to select the one BEST lettered option that answers each item. After you have selected the one lettered option that BEST answers each item, completely blacken the corresponding lettered circle for that item on Section A of the answer sheet. Please use only a #2 soft lead pencil.

Complex Case Highlight (Items 1–10)

A 35-year-old African-American man presents to you for his annual “physical exam.” He has no significantcomplaints but is concerned because his older sister was just diagnosed with type 2 diabetes. His father also had type 2 diabetes and died at age 54 of a myocardial infarction. He has two sons age 10 and 12, both L120% of ideal weight for their height. He is 6’0” and weighs 210 lbs (BMI 27 kg/m2); blood pressure is 138/88 mm Hg.

Current lab results for this patient are as follows:

K Items 1–6

For each numbered laboratory result (1–6), (if confirmed by repeat testing), select the one lettered diagnosis for thispatient’s problem (A, B, C, D) MOST likely associated with it. Each lettered diagnosis may be selected once, morethan once, or not at all.

(A) Impaired fasting glucose (IFG).

(B) Impaired glucose tolerance (IGT).

(C) Diabetes.

(D) None of the above.

1. Fasting plasma glucose 120 mg/dL.

2. 2-hour plasma glucose 190 mg/dL.

3. Fasting plasma glucose 130 mg/dL.

Total Cholesterol 210 mg/dL

Triglycerides 150 mg/dL

HDL-C 40 mg/dL

LDL-C 140 mg/dL

Glucose 96 mg/dL

American Diabetes Association Self-Assessment Program MCQs8

4. 2-hour plasma glucose 210 mg/dL after a dose of 75 g of anhydrous glucose dissolved in water.

5. Non-fasting glucose (he ate breakfast 4 hours before) 160 mg/dL.

6. The same patient but now with a history of polyuria and a non-fasting glucose of 210 mg/dL.

7. Which one of the following statements regarding this patient is CORRECT?

(A) According to ADA guidelines, since his fasting glucose is l100 mg/dL, testing should be repeated at 3-year intervals.

(B) According to the NCEP ATP III guidelines, he is at his LDL goal for his risk category.

(C) According to the JNC7, he does not require drug therapy for hypertension.

(D) According to the ADA guidelines, an oral glucose tolerance test (OGTT) is recommended to potentiallydiagnose diabetes in patients with a strong family history.

(E) According to ADA guidelines, his two sons should have their fasting plasma glucose measured andrepeated every 2 years.

Upon the recommendation of your patient, his sister comes to see you for treatment of her diabetes. His sister is 39 years old and was diagnosed with “diabetes” at a health fair in which she had a non-fasting glucose of 180 mg/dLapproximately 6 weeks ago. She has 5 children ages 10–20 years old. At birth, all her children weighed at least 9 lbs. She has a history of increased blood pressure for the past 5 years and has tried to watch her salt intake andlose weight but takes no medication. For the past 6 weeks, she has restricted her caloric intake and started walkingaround the block at least 5 times per week. She believes she has lost a “few pounds” but complains of feeling“sluggish” and she urinates more than usual. Her labs are as follows:

Total Cholesterol 180 mg/dL

Triglycerides 200 mg/dL

HDL-C 38 mg/dL

LDL-C 102 mg/dL

Fasting Glucose 170 mg/dL

A1C 7.2%

Creatinine 1.1 mg/dL

BUN 20

ALT 48 (normal l40)

AST 50 (normal l45)

Height 5’8”

Weight 170 lbs

Blood Pressure 138/88 mm Hg

9MCQs American Diabetes Association Self-Assessment Program

8. According to the ADA guidelines, which one of the following statements regarding the patient’s sister is CORRECT?

(A) Before initiating drug therapy, the patient should undergo an exercise stress test to rule out significantcoronary artery disease and, if normal, an aerobic exercise program should be prescribed for at least 60 minutes/day 5 times per week.

(B) Start a low-carbohydrate diet (restricting total carbohydrates to l130 g/day) to induce short-term weightloss and thereby improve glycemic control.

(C) Start metformin 1000 mg/day in conjunction with a weight reduction meal plan and an exercise program.

(D) Start therapy with a sulfonylurea because she has symptomatic hyperglycemia and this medication willresult in more rapid reduction in her elevated glucose level.

(E) Start with a glitazone because she has a very low HDL (l50 mg/dL) for a woman.

9. In addition to treatment for hyperglycemia, which one of the following statements is CORRECT regarding the management of her other cardiovascular risk factors according to AHA recommendations?

(A) Since she has a systolic blood pressure of 130–139 mm Hg and a diastolic blood pressure of 80–89 mm Hg and has failed lifestyle and behavior therapy, she should be treated with pharmacologicalagents that block the rennin-angiotensin system.

(B) Since she is less than 40 years of age, statin therapy is not recommended.

(C) Since she does not have overt CVD, her LDL goal is l130 mg/dL rather than l100 mg/dL.

(D) For a women with diabetes, her triglyceride goal is l100 mg/dL and her HDL goal is L40 mg/dL.

(E) A fibrate is recommended as the drug of first choice to treat her dyslipidemia because she has elevatedtriglycerides.

10. For a patient with newly-diagnosed type 2 diabetes, which one of the following ancillary tests is recommended?

(A) Urinary evaluation for microalbuminuria.

(B) A dilated and comprehensive eye examination by an ophthalmologist or optometrist.

(C) Screening for distal symmetric polyneuropathy.

(D) Comprehensive foot exam.

(E) All of the above.

K Items 11–12

A 55-year-old African-American female school teacher whose mother and father both developed type 2 diabetes in their 50’s presents to you for an evaluation to determine her risk for developing diabetes mellitus. She has 4 children and during the pregnancy of her fourth child, she developed gestational diabetes. She is a non-smokerand sedentary. She is 5’5”, weighs 200 lbs, and blood pressure is 128/78 mm Hg. Fasting labs are as follows:

Total Cholesterol 200 mg/dL

Triglycerides 100 mg/dL

HDL-C 50 mg/dL

LDL-C 130 mg/dL

Glucose 110 mg/dL

American Diabetes Association Self-Assessment Program MCQs10

She takes no medications including aspirin. She brings with her a Personal Health Decisions report fromwww.diabetes.org/diabetesphd, which is an appraisal on the ADA website for assessing the risk of developingdiabetes, heart disease, or a stroke over the next 30 years.

11. Based on the Personal Health Decisions (Diabetes PHD) appraisal, she has a 38% chance of developing type2 diabetes. If she loses 10% of her body weight (approximately 20 lbs), her risk of developing type 2 diabeteswould be which one of the following?

(A) Modified to l10%.

(B) Modified to l11–20%.

(C) Modified to l21–30%.

(D) Her risk remains L35% because she is still significantly overweight.

(E) Due to her history of gestational diabetes, weight loss would be ineffective in modifying her risk of developing type 2 diabetes.

12. According to the ADA Guidelines for the prevention or delay of type 2 diabetes, which one of the followingnext steps would be MOST appropriate?

(A) Initiate therapy with metformin simultaneously with a weight reduction meal plan or exercise program.

(B) Perform a 2-hour oral glucose tolerance test and only initiate metformin if her 2-hour post-prandialglucose is greater than 180 mg/dL.

(C) Instruct her on a weight reduction meal plan with exercise. If she fails to lose weight and her fastingglucose remains below 126 mg/dL, medication remains inappropriate.

(D) Instruct her on a weight reduction meal plan program with exercise and consider initiation of metforminin 6–12 months if she has not lost significant amounts of weight and her fasting glucose remains greaterthan 100 mg/dL.

(E) Instruct the patient on a weight reduction meal plan with exercise with the addition of a weight lossmedication, such as orlistat.

K Item 13

A 40-year-old Hispanic woman with newly-diagnosed type 2 diabetes mellitus has been on a weight reduction meal plan for the past 3 months and has lost 5 lbs. She is on simvastatin 20 mg, enalapril 10 mg, and aspirin 81 mg.Her labs are as follows:

Total Cholesterol 160 mg/dL

Triglycerides 200 mg/dL

HDL-C 40 mg/dL

LDL-C 70 mg/dL

Glucose 180 mg/dL

A1C 9.0%

Height 5’1”

Weight 150 lbs

Waist Circumference 37”

Blood Pressure 128/80 mm Hg

11MCQs American Diabetes Association Self-Assessment Program

13. According to the American Diabetes Association (ADA) treatment algorithm for type 2 diabetes,the MOST appropriate treatment option for this patient is which one of the following?

(A) Since she has lost 5 lbs, oral hypoglycemic agents should be withheld until weight loss treatment has been maximized.

(B) Initiate metformin therapy as the first-line treatment for type 2 diabetes.

(C) Since she has significant glucose elevation, a sulfonylurea should be initiated because these oralhypoglycemic agents are the most potent in treating hyperglycemia.

(D) Because she has elevated triglyceride and low HDL levels, a thiazolidinedione (TZD) should be initiated.

(E) She should receive treatment with sitagliptin because it has a low side-effect profile and is a category Bdrug for women of child-bearing potential.

K Items 14–18

For each numbered clinical scenario or pharmacologic effect (14–18), select the one MOST appropriate letteredPPAR agonist treatment option associated (A, B, C, D, E). Each lettered PPAR agonist treatment option may beselected once, more than once, or not at all.

(A) Fenofibrate.

(B) Gemfibrozil.

(C) Thiazolidinedione.

(D) None of the above.

(E) All of the above.

14. Contraindicated in a man with cardiovascular disease and NYHA Class I heart failure.

15. Inhibits statin glucuronidation leading to elevated statin levels.

16. Dosage adjustment is necessary in patients with moderate to severe renal impairment.

17. Not contraindicated in a woman with diabetes and fatty liver disease in whom the ALT is 75 U/L (normal l40) and the AST 46 U/L (normal l36).

18. Causes a fall in the hematocrit.

K Items 19–23

For each numbered physiological or pathophysiological effect (19–23), select the one MOST accurate letteredadipocytokine (A, B, C, D, E) associated with that effect. Each lettered adipocytokine may be selected once,more than once, or not at all.

(A) Adiponectin.

(B) TNF-α (tumor necrosis factor-alpha).

(C) Free fatty acids.

(D) Both TNF-α and free fatty acids.

(E) PAI-1 (plasminogen activator inhibitor-1).

American Diabetes Association Self-Assessment Program MCQs12

19. Increases insulin sensitivity.

20. Inhibits monocyte adhesion to vascular wall endothelium.

21. Causes hyperinsulinemia by interfering with hepatic insulin degradation.

22. Contributes to insulin resistance.

23. Localizes in the plaque where it is thought to promote coagulation.

24. Which one of the following statements is CORRECT regarding the therapy of patients with type 2 diabetesmellitus with biguanides?

(A) They will be the drug of choice in patients with compromised left ventricular function due to their lowrisk of hypoglycemia.

(B) They should not be used in patients with significant renal impairment.

(C) They should not be used in combination with insulin due to the increased risk for side-effects.

(D) They lose efficacy when combined with a thiazolidinedione.

(E) They are best tolerated and most effective when taken on an empty stomach.

25. Which one of the following mechanisms MOST likely contributes to accelerated vascular disease associatedwith insulin resistance?

(A) The direct effect of elevated insulin levels on the vessel wall.

(B) A pro-inflammatory state.

(C) Accelerated fatty acid oxidation in vascular smooth muscle cells.

(D) Reduced plasminogen activator inhibitor-1 (PAI-1) levels.

(E) Reduced homocysteine levels.

26. Which one of the following statements is CORRECT regarding home monitoring of blood glucose?

(A) It is only useful for patients on insulin.

(B) In the most aggressively managed patients it should be done every 6 hours.

(C) It is generally not necessary in patients with type 2 diabetes mellitus.

(D) Measuring a fasting blood glucose is most helpful for adjusting a basal insulin dose.

(E) None of the above.

13MCQs American Diabetes Association Self-Assessment Program

K Items 27–31

For each numbered oral hypoglycemic agent (27–31), select the one lettered mechanism of action (A, B, C, D, E) which is MOST important for its efficacy in lowering blood glucose. Each lettered mechanism of action may beselected only once.

(A) Increases insulin secretion.

(B) Delays glucose absorption in the intestine.

(C) Suppresses glucose production in the liver.

(D) Increases glucose uptake into muscle.

(E) Inhibits the enzyme that degrades GLP-1 (glucagon-like protein-1).

27. Sulfonylureas.

28. Thiazolidinediones.

29. Biguanides.

30. Alpha-glucosidase inhibitors.

31. Sitagliptin.

K Items 32–35

For each numbered undesirable side-effect (32–35), select the one lettered class of oral hypoglycemic agent (A, B, C, D, E) MOST closely associated with it. Each lettered class of oral hypoglycemic agent may be selected once, more than once, or not at all.

(A) Sulfonylureas.

(B) Thiazolidinediones.

(C) Biguanides.

(D) Alpha-glucosidase inhibitors.

(E) None of the above.

32. Increased fluid retention and edema.

33. Nausea; lactic acidosis.

34. Hypoglycemia.

35. Gas, bloating.

American Diabetes Association Self-Assessment Program MCQs14

36. A female patient with type 2 diabetes mellitus, weighing 300 lbs, asks you about treatment options but is afraid of using medication for the treatment of diabetes. Which one of the following is the MOSTeffective method of achieving significant weight loss in patients with diabetes mellitus without using antihyperglycemia medications?

(A) Diets emphasizing fewer calories.

(B) Increased physical activity.

(C) Diets emphasizing a reduction in saturated fats.

(D) Bariatric surgery.

(E) Medications for weight loss.

37. An overweight man with type 2 diabetes mellitus and peripheral neuropathy asks you about starting anexercise program to lose weight. Which one of the following is NOT a beneficial effect of incorporatingincreased physical activity and weight loss into the regimen of this patient?

(A) Reduction in blood glucose.

(B) Reduction in blood pressure.

(C) Improvement in lipid profile.

(D) Improved sense of control over the illness.

(E) Will achieve less injuries related to neuropathy.

38. Metformin is often selected when starting a patient with newly-diagnosed diabetes on medication for the firsttime. Which one of the following is NOT an advantage with metformin over other medications?

(A) It rarely causes hypoglycemia.

(B) It rarely induces weight gain.

(C) It is associated with no serious side-effects.

(D) It is generally inexpensive.

(E) When dosed effectively it will usually lower A1C more than 1%.

39. Sulfonylureas are frequently prescribed as part of the regimen for patients with type 2 diabetes.Which one of the following concerns about sulfonylureas is INCORRECT?

(A) Have a tendency to cause weight gain.

(B) May cause hypoglycemia, especially the longer-acting agents.

(C) Have been consistently shown to increase mortality from cardiovascular disease.

(D) May interact with other sulfa drugs or glinides to increase risk of hypoglycemia.

(E) May cause a rash.

15MCQs American Diabetes Association Self-Assessment Program

40. The glinides (e.g., repaglinide and nateglinide) are a recommended substitute for sulfonylureas for use as monotherapy in patients with type 2 diabetes mellitus with which one of the following?

(A) Congestive heart failure.

(B) Liver disease.

(C) Renal insufficiency.

(D) Associated hypertriglyceridemia.

K Items 41–45

For each numbered anti-hyperglycemic agent (41–45), select one lettered primary mechanism of action (A, B, C, D, E) associated with it. Each lettered primary mechanism of action may be selected only once.

(A) Reduces gluconeogenesis in the liver.

(B) Stimulates post-prandial insulin secretion.

(C) Delays the absorption of carbohydrates in the gut.

(D) Enhances insulin sensitivity in the peripheral tissues.

(E) Inhibits the breakdown of incretins.

41. Metformin.

42. Glitazones.

43. Alpha-glucosidase inhibitors.

44. Glinides.

45. Sitagliptin.

K Items 46–48

A 50-year-old woman with type 2 diabetes mellitus is seen in the office with a blood pressure of 150/90 mm Hgand the lipid profile shown below. She has a normal amount of albumin in her urine and normal renal function.

Total Cholesterol 215 mg/dL

Triglycerides 120 mg/dL

HDL-C 51 mg/dL

LDL-C 140 mg/dL

American Diabetes Association Self-Assessment Program MCQs16

46. If her doctor elects to start her on medication for her blood pressure, which one of the following classes of antihypertensive agents would be BEST to reduce her chance of progression to microalbuminuria?

(A) Alpha blockers.

(B) Diuretics.

(C) ACE inhibitors.

(D) Beta blockers.

(E) Calcium channel blockers.

47. She is reluctant to take a statin. Which one of the following statements is INCORRECT regarding statins and the prevention of CVD?

(A) Statins have been shown to reduce the chances of a CV event in individuals that have no known CVD (primary prevention).

(B) All the large published studies which evaluated statins’ ability to prevent all cause mortality have shown a benefit.

(C) Statins have been shown to reduce the chances of a second CV event in individuals that have known CVD (secondary prevention).

(D) Statins have been shown to reduce the chances of having a CV event whether the LDL level is L116 mg/dL or l116 mg/dL

(E) The American Diabetes Association recommends all patients with type 2 diabetes mellitus reduce LDL levels to 100 mg/dL or lower.

48. An alpha-glucosidase inhibitor, such as acarbose would be LEAST appropriate for the treatment of a patientwith type 2 diabetes mellitus and which one of the following clinical characteristics?

(A) A1C level of 7.5 mg/dL with kidney disease.

(B) A1C level of 7.5 mg/dL with liver disease.

(C) A1C level of 8.0 mg/dL with a tendency toward hypoglycemia.

(D) A1C level of 12.0 mg/dL with low tolerance for side-effects.

(E) A1C level of 7.2 mg/dL with concerns about systemic side-effects.

49. Insulin should be considered, along with lifestyle modifications, as the initial therapy in a newly-diagnosedpatient with type 2 diabetes mellitus in all of the following situations EXCEPT:

(A) Fasting blood glucose level L250 mg/dL.

(B) A1C L10%.

(C) BMI l27 kg/m2.

(D) Presence of ketonuria.

(E) Symptomatic hyperglycemia (polyuria, polydypsia).

17MCQs American Diabetes Association Self-Assessment Program

50. A 43-year-old overweight hypertensive woman is newly-diagnosed with type 2 diabetes mellitus, with a A1Cof 8.5%. Which one of the following is the BEST choice for initial therapy in addition to weight reduction?

(A) Metformin.

(B) A sulfonylurea.

(C) A glitazone.

(D) Inhaled insulin.

(E) Sitagliptin.

51. When initiating insulin therapy in a patient with type 2 diabetes, which one of the following approaches is recommended?

(A) Start with a bedtime dose of an intermediate-acting form or a once daily, long-acting form.

(B) Start with a 2-shot regimen of a long-acting form.

(C) Start with a 3-shot regimen of a short-acting form.

(D) Start with a higher dose and titrate down.

(E) Make dosing changes infrequently to give time for the patient to adjust to the dose.

52. A 48-year-old man with type 2 diabetes, treated with a basal dose of long-acting insulin and oral medications,is struggling to control his post-prandial blood glucose. When adding rapid-acting insulin with meals to abasal insulin dosage, it is important to remember which one of the following additional adjustments?

(A) Reduce the basal insulin dose to avoid hypoglycemia.

(B) Discontinue any insulin secretogogue, like a sulfonylurea or glinide.

(C) Increase the dose of the insulin secretogogues to enhance the effect of the injected insulin.

(D) Be sure to add an ACE inhibitor to improve renal function.

(E) Reduce the total carbohydrate intake by 30% to avoid hyperglycemia.

K Items 53–54

A 51-year-old woman with type 2 diabetes has been treated for 3 months with lifestyle modifications and metformin at 1000 mg b.i.d., but her A1C level is still L7%.

53. Which one of the following steps would be LEAST appropriate for her at this time?

(A) Add a glitazone (rosiglitazone or pioglitazone).

(B) Add glipizide.

(C) Add a single dose of basal insulin.

(D) Increase the metformin to 1000 mg t.i.d.

(E) Add glyburide.

American Diabetes Association Self-Assessment Program MCQs18

54. The patient is reluctant to intensify her therapy because she feels fine. In discussing the benefits of reducingher A1C level, you consider how much benefit she can derive from additional reduction. According to the United Kingdom Prospective Diabetes Study (UKPDS), for every 1% reduction in A1C level in theaverage patient with type 2 diabetes, which one of the following results would NOT be expected?

(A) A 21% decrease in any diabetes-related endpoint.

(B) A 14% decrease in the risk of myocardial infarction.

(C) A 12% decrease in the risk of stroke.

(D) A 37 % decrease in the risk of microvascular disease.

(E) A 42% decrease in the risk of heart failure.

55. Which one of the following vaccines is recommended for all patients with diabetes between the ages of 18–64 years?

(A) Meningococcal.

(B) Pneumococcal once and yearly influenza.

(C) Hepatitis A.

(D) Hepatitis B.

(E) Pneumococcal every 5 years and influenza yearly.

56. On average, patients with type 2 diabetes mellitus have lost approximately what percentage of their pancreaticbeta-cell (β-cell) function by the time their diabetes is diagnosed?

(A) 20%.

(B) 35%.

(C) 50%.

(D) 70%.

(E) 90%.

57. Which one of the following is NOT commonly associated with hypoglycemia?

(A) Anxiety.

(B) Tachycardia.

(C) Shakiness.

(D) Death.

(E) Diaphoresis.

19MCQs American Diabetes Association Self-Assessment Program

K Items 58–59

The pathophysiology of type 2 diabetes mellitus requires both insulin resistance and pancreatic β-cell dysfunction.The major targets for insulin are the liver, skeletal muscle, and adipose tissue.

58. Which one of the following statements is INCORRECT regarding insulin resistance in patients with type 2 diabetes mellitus?

(A) The normal suppression of gluconeogenesis by insulin in the liver is impaired.

(B) The normal uptake of glucose by skeletal muscle is impaired.

(C) The normal glucose disposal, which includes glucose oxidation and glycogen synthesis, by skeletalmuscle, is impaired.

(D) Excess free fatty acids (FFAs) impair β-cell function.

(E) The normal degradation of triglycerides into FFAs by adipose tissue is impaired.

59. Which one of the following is NOT a major factor in pancreatic β-cell dysfunction in patients with type 2 diabetes mellitus?

(A) Environmental factors.

(B) Genetic factors.

(C) Lipotoxicity.

(D) Glucotoxicity.

(E) Insulin toxicity.

60. Which one of the following conditions depicted pictorially is NOT associated with type 2 diabetes mellitus?

(A) (B)

(C) (D)

American Diabetes Association Self-Assessment Program MCQs20

Management of a patient with type 2 diabetes (Items 61–71)

A 28-year-old African-American man presented to his primary care physician’s office with complaints of knee pain.In college, he played guard on the football team. He had anterior cruciate ligament surgery after college, stoppedregular exercise, and gradually increased weight.

Past History: no other surgery.Family History: positive for type 2 diabetes mellitus, hypertension.Social History: negative for smoking, no drug abuse, occasional alcohol intake.Review of symptoms: non-contributory.

61. The patient asks how to prevent overt diabetes. Which one of the following therapies has NOT been proven to prevent diabetes in patients with impaired glucose tolerance?

(A) Lifestyle changes.

(B) Metformin.

(C) Acarbose.

(D) Rosiglitazone.

(E) Ramipril.

62. Diet should never be forgotten as part of the care of patients with diabetes. The optimal mix of macronutrientsis not clear; in fact, a diet consisting of foods which the patient usually eats, modified to fit basic principles,will ensure compliance. In a patient, such as this 28-year-old African-American man with impaired glucosetolerance or type 2 diabetes, which one of the following diets is NOT recommended?

(A) A diet that includes sugar alcohols and non-nutritive sweeteners in FDA-approved quantities.

(B) A diet that includes fruits, vegetables, fiber, whole grain, legumes and low fat meals.

(C) A diet that includes fiber.

(D) A diet that includes l20 g of carbohydrate foods daily.

(E) A diet that includes controlled quantities of sucrose-containing foods.

Triglycerides 152 mg/dL

HDL-C 38 mg/dL

LDL-C 110 mg/dL

Post-prandial Glucoses (PPG) 144 mg/dL & 154 mg/dL

A1C 5.6%

Blood Pressure 130/82 mm Hg

Heart Rate 76 BPM

Respirations 14

BMI 34 kg/m2

Fundi normal

Pulmonary, cardiac, abdominal & neurological exams normal

21MCQs American Diabetes Association Self-Assessment Program

63. Which one of the following is a characteristic seen in some African-Americans with new-onset type 2 diabetes?

(A) Ability to stop insulin therapy soon after presentation with diabetic ketoacidosis (DKA).

(B) A particular responsiveness to a thiazolidinedione (TZD).

(C) Commonly have anti-islet cell antibodies.

(D) Low incidence of family history of diabetes.

(E) Associated with sickle cell trait.

64. A generally acceptable ideal goal of treatment for this patient might include each of the following EXCEPT:

(A) A1C l7%.

(B) A1C l6% without undue hypoglycemia.

(C) Fasting blood glucose l140 mg/dL.

(D) Peak post-prandial blood glucose l180 mg/dL.

65. This patient’s A1C level on lifestyle therapy alone was maintained at ~5.8% for about a year, but graduallyrose to p6.5%, and on return to the practice was 7.2%. It was elected to prescribe metformin. According toADA guidelines, metformin should be started at the time of diagnosis because of which one of the following?

(A) Effectiveness in decreasing A1C.

(B) Low cost.

(C) Acceptable low risk of side-effects.

(D) Proven outcome benefits.

(E) All of the above.

66. Which one of the following is NOT an effect of therapy with metformin?

(A) It reduces hepatic glucose output.

(B) It reduces hepatic gluconeogenesis.

(C) It reduces insulin levels.

(D) It reduces fasting blood glucose levels.

(E) It increases C-peptide levels.

67. The patient did well for about 2 years on metformin. However the AIC again increased to 7.2% and pioglitazone was added to metformin. His A1C decreased to 6.4%. However, his A1C increased again and it was elected to add a third agent when it reached 7.1%. Which one of the following is NOT a logicalchoice for a third drug added to this patient’s treatment?

(A) Glipizide.

(B) Rosiglitazone.

(C) Glargine.

(D) Exenatide.

(E) Sitagliptin.

American Diabetes Association Self-Assessment Program MCQs22

68. Which one of the following is NOT an abnormality resulting from increased visceral fat in patients with the metabolic syndrome?

(A) Increased myocyte insulin resistance.

(B) Increased liver insulin resistance.

(C) Increased adiponectin level.

(D) Increased likelihood for the onset of type 2 diabetes.

(E) Decreased β-cell function.

69. Which one of the following statements is INCORRECT regarding post-prandial glycemia?

(A) It is the earliest detectable glycemic abnormality in the majority of patients.

(B) Convincing data exists that treating it will reduce cardiovascular outcomes or other complications of diabetes.

(C) It contributes to the A1C level of 6.5% to a greater extent than does fasting blood glucose.

(D) It is associated with an increased risk of microvascular complications.

(E) It is associated with an increased risk of macrovascular complications.

70. Primary data supporting the use of analog insulin does NOT include which one of the following?

(A) It mimics normal physiology.

(B) It decreases overnight hypoglycemia.

(C) It reduces A1C level more than does human insulin.

(D) Patient lifestyle will control the disease therapy rather than the disease therapy controlling the lifestyle.

(E) It decreases late post-prandial hypoglycemia.

71. Which one of the following is NOT a likely mechanism for complications in diabetes mellitus?

(A) Increased pentose-phosphate shunt flux.

(B) Increased flux in aldose reductase pathway.

(C) Increased advanced glycosylation end product production.

(D) Increased hexosamine pathway flux.

(E) Increased oxidative stress in mitochondria.

72. Which one of the following statements about treatments of hypertension in patients with type 2 diabetes is INCORRECT?

(A) Aim for blood pressure under 130/80 mm Hg.

(B) Include an ACE inhibitor or an ARB if possible.

(C) Often requires combination therapy.

(D) Must absolutely contraindicate hydrochlorothiazide (HCTZ) because it may increase blood glucose.

(E) Beta blockade may increase the risk of hypoglycemic unawareness.

23MCQs American Diabetes Association Self-Assessment Program

K Items 73–75

A logical choice of combination therapy with metformin should depend on individual characteristics of patientswith type 2 diabetes. For each numbered patient being treated with metformin (73–75), select the one letteredadditional agent in combination therapy (A, B, C) most appropriate for that patient. Each lettered additional agentin combination therapy may be selected only once.

(A) Insulin.

(B) Exenatide.

(C) Sitagliptin.

73. A patient with a BMI L35 kg/m2.

74. A patient with A1C L9.6%.

75. A patient who is a long-distance truck driver and fears injections.

K Notes

American Diabetes Association Self-Assessment Program 24

25Appendix American Diabetes Association Guidelines and Treatment Algorithms At-A-Glance

AppendixI. American Diabetes Association Guidelines

and Treatment Algorithms At-a-Glance

26 American Diabetes Association Guidelines and Treatment Algorithms At-A-Glance Appendix

Definition and Description of Diabetes MellitusDiabetes mellitus is a group of metabolic diseases characterized by hyperglycemia resulting from defects in insulinsecretion, insulin action, or both. The chronic hyperglycemia of diabetes is associated with long-term damage,dysfunction, and failure of various organs, especially the eyes, kidneys, nerves, heart, and blood vessels.

I. Diagnosis of impaired glucose tolerance (IGT) and impaired fasting glucose (IFG)This group is defined as having fasting plasma glucose (FPG) levels M100 mg/dL (5.6 mmol/L) but l126 mg/dL (7.0 mmol/L) or 2-h values in the oral glucose tolerance test (OGTT) of M140 mg/dL (7.8 mmol/L)but l200 mg/dL (11.1 mmol/L). Thus, the categories of FPG values are as follows:

K FPG l100 mg/dL (5.6 mmol/L) normal fasting glucose;

K FPG 100–125 mg/dL (5.6–6.9 mmol/L) = IFG;

K FPG M126 mg/dL (7.0 mmol/L) provisional diagnosis of diabetes (the diagnosis must be confirmed,as described below).

The corresponding categories when the OGTT is used are the following:

K 2-h post load glucose l140 mg/dL (7.8 mmol/L) normal glucose tolerance;

K 2-h post load glucose 140–199 mg/dL (7.8–11.1 mmol/L) = IGT;

K 2-h post load glucose M200 mg/dL (11.1 mmol/L) provisional diagnosis of diabetes (the diagnosis must be confirmed, as described below).

Patients with IFG and/or IGT are now referred to as having “pre-diabetes” indicating the relatively high risk for development of diabetes in these patients.

Table 1. Treatment recommendation for individuals with IFG, IGT, or both.

* Metformin 850 mg twice/day.

Population Treatment

IFG or IGT. Lifestyle modification (i.e., 5–10% weight loss and moderateintensity physical activity p30 min/day).

Individuals with IFG and IGT and any of the following: Lifestyle modification (as above) and/or metformin*.K L60 years of age.K BMI M35 kg/m2.K Family history of diabetes in first-degree relatives.K Elevated triglycerides.K Reduced HDL-C.K Hypertension.K A1C L6.0%.

27Appendix American Diabetes Association Guidelines and Treatment Algorithms At-A-Glance

II. Diagnosis of type 2 diabetesTable 2. Criteria for testing for diabetes in asymptomatic adult individuals.

* May not be correct for all ethnic groups.

Table 3. Testing for type 2 diabetes in children.

Clinical judgment should be used to test for diabetes in high-risk patients who do not meet these criteria.

Table 4. Criteria for the diagnosis of diabetes mellitus.

In the absence of unequivocal hyperglycemia, these criteria should be confirmed by repeat testing on a different day. The third measure (OGTT) is not recommended for routine clinical use.

1. Symptoms of diabetes plus casual plasma glucose concentration M200 mg/dL (11.1 mmol/L).1). Casual is defined as any time of day without regard to time since last meal. The classic symptoms of diabetes includepolyuria, polydipsia, and unexplained weight loss.

OR

2. FPG M126 mg/dL (7.0 mmol/L). Fasting is defined as no caloric intake for at least 8 hours.

OR

3. 2-h post load glucose M200 mg/dL (11.1 mmol/L) during an OGTT. The test should be performed as described by the World Health Organization, using a glucose load containing the equivalent of 75 g anhydrous glucose dissolved in water.

Criteria

K Overweight (BMI L85th percentile for age and sex, weight for height L85th percentile, or weight L120% of ideal for height).

Plus any two of the following risk factors:

K Family history of type 2 diabetes in first- or second- degree relative.K Race/ethnicity (Native American, African-American, Latino, Asian American, Pacific Islander).K Signs of insulin resistance or conditions associated with insulin resistance (acanthosis nigricans, hypertension,

dyslipidemia, or PCOS).K Maternal history of diabetes or GDM.

Age of initiation: age 10 years or at onset of puberty, if puberty occurs at a younger age.Frequency: every 2 years.Test: FPG preferred.

1. Testing for diabetes should be considered in all individuals at age 45 years and above, particularly in those with a BMI M25 kg/m2*, and, if normal, should be repeated at 3-year intervals.

2. Testing should be considered at a younger age or be carried out more frequently in individuals who are overweight (BMI M25 kg/m2*) and have additional risk factors:K Are habitually physically inactive.K Have a first-degree relative with diabetes.K Are members of a high-risk ethnic population (e.g., African-American, Latino, Native American, Asian American,

Pacific Islander).K Have delivered a baby weighing L9 lbs or have been diagnosed with GDM.K Are hypertensive (M140/90 mm Hg).K Have an HDL-C level l35 mg/dL (0.90 mmol/L) and/or a triglyceride level L250 mg/dL (2.82 mmol/L).K Have PCOS (polycystic ovary syndrome).K On previous testing, had IGT or IFG.K Have other clinical conditions associated with insulin resistance (e.g., PCOS or acanthosis nigricans).K Have a history of vascular disease.

28 American Diabetes Association Guidelines and Treatment Algorithms At-A-Glance Appendix

III. Summary of recommendations for adults with diabetes.

* Referenced to a non-diabetic range of 4.0–6.0% using a DCCT-based assay.† Post-prandial glucose measurements should be made 1–2 hours after the beginning of the meal, generally peak levels in patients with diabetes.‡ Current NCEP/ATP III guidelines suggest that in patients with triglycerides M200 mg/dL, the “non-HDL-C” (total cholesterol minus HDL) be utilized.

The goal is m130 mg/dL (121).§ For women, it has been suggested that the HDL-C goal be increased by 10 mg/dL.

Table 5. Detailed diabetes history and evaluation.

Adapted from American Diabetes Association Clinical Practice Recommendations, 2006. Diabetes Care 2006;29(suppl. 1):S9.

Past medical history should include:

K Prior glucose values including A1C, symptoms, treatments, and history of self-monitoring.K History of complications: hypoglycemia and/or ketoacidosis – including frequency, severity, and causes.K Self-management, self-monitoring education, attitudes, health beliefs.K Mood disorder.K Prior or current infections, particularly skin, foot, dental, and genitourinary.K Current microvascular symptoms and complications (eye, kidney, nerves).K Current macrovascular symptoms and complications (evaluation of cardiovascular risk factors and cardiovascular disease).K Sexual dysfunction.K Review of past/present lifestyle modification

k Eating patterns k Smoking historyk Diet history k Alcohol historyk Weight history k Controlled substancesk Exercise history

K Other medications – evaluate for effect on blood glucose.K Family history of diabetes and CVD.

Referrals

K Eye exam. K Diabetes educator.K Family planning for women of reproductive age. K Foot specialist.K Dietitian for medical nutrition therapy (MNT). K Dentist.

Glycemic Control

A1C l7.0%*

Preprandial capillary plasma glucose 90–130 mg/dL (5.0–7.2 mmol/L)

Peak post-prandial capillary plasma glucose† l180 mg/dL (l10.0 mmol/L)

Blood pressure l130/80 mm Hg

Lipids‡

LDL-C l100 mg/dL (l2.6 mmol/L)

Triglycerides l150 mg/dL (l1.7 mmol/L)

HDL-C L40 mg/dL (L1.0 mmol/L)§

Key concepts in setting glycemic goals:K A1C is the primary target for glycemic control.K Goals should be individualized.K Certain populations (children, pregnant women, and elderly) require special considerations.K More stringent glycemic goals (i.e., a normal A1C, l6%) may further reduce complications at the cost

of increased risk of hypoglycemia.K Less intensive glycemic goals may be indicated in patients with severe or frequent hypoglycemia.K Post-prandial glucose may be targeted if A1C goals are not met despite reaching preprandial glucose goals.

29Appendix American Diabetes Association Guidelines and Treatment Algorithms At-A-Glance

Table 6. Follow-up testing recommendations with diabetes.

Adapted from American Diabetes Association Clinical Practice Recommendations, 2006. Diabetes Care 2006;29(suppl. 1):S4–42.

Potential Complications Test Frequency of Evaluation

Hyperglycemia A1C Measure A1C 2 times/year if at goal, quarterly if therapy changed, or not at goal

Fasting blood glucose Varies depending on prescription regime

Self-monitoring of blood glucose Frequency/timing should be individualizedto facilitate reaching goals

Hypertension Blood pressure Measure at every visit

Autonomic Neuropathy Sitting and standing blood pressure Appropriate patients/each visit(Orthostatic Hypotension)

Dyslipidemia Lipid panel Test annually – More often if not at goal or when changing therapy

Foot Care Quantitative somato-sensory threshold Perform a visual inspection at eachtest using Semmes-Weinstein 5.07 (10 g) routine visit for people with neuropathy.monofilament, tuning fork, palpation, and visual inspection

Nephropathy Serum creatinine Annually

Albumin/Creatinine ratio Type 2 diabetes mellitus – Annual screening

(Random spot urine) Type 1 diabetes mellitus – Annual screening with duration of diabetes M5 years

Type 1 & 2 diabetes mellitus – 2 of 3 specimens within a 3–6 month period should be abnormal before consideringa new diagnosis

Retinopathy Comprehensive eye exam Type 2 diabetes mellitus – Annual dilated &comprehensive exam

Type 1 diabetes mellitus – Initial dilated &comprehensive exam within 5 years of onset;annual exams thereafter

Periodontal disease (common Dental evaluation Annuallyin type 2 diabetes mellitus)

Influenza and pneumonia Immunization Annual influenza vaccination

Annual comprehensive foot exam

30 American Diabetes Association Guidelines and Treatment Algorithms At-A-Glance Appendix

Figure 1. Algorithm for the metabolic management of type 2 diabetes. Reinforce lifestyle intervention at every visit.

* Check A1C every 3 months until l7% and then at least every 6 months.† Although three oral agents can be used, initiation and intensification of insulin therapy is preferred based on effectiveness and expense.

Each anti-diabetic medication class possesses differences with regard to contraindications, adverse effects andmechanisms of action that may dictate which medication is the best for an individual patient (Table 7). Sulfony-lurea therapy may be most useful in thin patients with insulinopenia, while metformin may be most useful in obese patients with dyslipidemia. The α-glucosidase inhibitors and meglitinides may be most useful in patients with exaggerated post-prandial increases in blood glucose, however the α-glucosidase inhibitors can be used withsulfonylureas while the meglitinides can be used with metformin. The thiazolidinediones may be most useful inpatients with insulin resistance or azotemia. Insulin may be useful in patients who will not be able to reach theirglycemic goal with the use of the other adjunct therapies just mentioned.

Exenatide injection is the first in a new class of drugs for the treatment of type 2 diabetes called incretin mimetics.Exenatide increases glucose-dependent insulin secretion, suppresses inappropriately elevated glucagon secretion,and slows gastric emptying. Exenatide is indicated as adjunctive therapy to improve glycemic control in patientswith type 2 diabetes who are taking sulfonylurea, metformin, a TZD or combinations of these agents but have notachieved adequate glycemic control.

Lifestyle Intervention + Metformin

Diagnosis

Add Sulfonylurea(Least expensive)

Add Glitazone(No hypoglycemia)

Add Basal Insulin(Most effective)

Yes*No A1C M7%

Yes*No A1C M7% Yes*No A1C M7% Yes*No A1C M7%

Intensify Insulin Add Glitazone† Add Basal Insulin Add Sulfonylurea†

Yes*No A1C M7%Yes*No A1C M7%

Add Basal or Intensify Insulin

Intensive Insulin + Metformin +/- Glitazone

31Appendix American Diabetes Association Guidelines and Treatment Algorithms At-A-Glance

Table 7. Drug choices based on drug & patient characteristics

Other Drug Information

Primary failure rate

20% 20% 20% 20% No data

Cost xx xxx Xgeneric, XXbrand xx xxx xxx xx

? Ø long termcomplications

Ø AGE products in vitroØ endothelialdysfunction

Ø endothelialdysfunctionPreserve beta cell function.Only for Pio= advantageous for lipid profile

Metformin Thiazolidinedione SulfonylureaRepaglinide/Nateglinide* Exenatide

Sitagliptin(Dpp-4inhibtor)

Acarbose/Miglitol

Dosingfrequency

2–3x daily Piogli- Rosigli-tazone: tazone:1x daily 2x daily

Glime- Glipizide/pramide: Glyburide:1x daily 2x daily

* 1 with each meal

Inject beforebreakfast and dinner

1x daily 3x daily with meals

Speed of actionFBS/ppg

Weeks/weeks Weeks Day Day Day Day Months

EfficacyPost-prandialFBSHgA1

xxxxxxxx

xxxxxxxx

xxxxxxxx

xxx xxxxx xxxx x

xx

xx

xxxxx

xxxxx

Concern/side-effects

GI upset, lacticacidosis: avoid inpatients with CHF,liver disease, acuteillness, otheracidosis, Cr >1.4,hospitalizedpatients and hold 2 days after IV contrast untilCreatinine stable

EdemaAvoid with no saltdiet & avoid NSAIDChannel blocker,COX 2 inhibitorsCHF 1%

wt gain +++ wt gain ++/+ Nausea –Avoid by stopeating whenfeel full

None GI side- effectsTreat hypowith glucose

Hypoglycemicrisk when used alone

None None + + + + +/ + None None None

Monitor for Creatinine &creatinine clearance

Edema/Heart Failure

Hypoglycemia Hypoglycemia Hypoglcemiaif use withSU/insulin

Hypglocemiaif use withSU/insulin

LFTs

Weight Ø OR NC ≠ OR NC ≠ ≠ Decrease Neutral NC

LipidsHDLLDLTG

NCØ ØØ Ø

≠ ≠≠ ≠Ø Decrease

bothNCNCØ OR NC Decrease Decrease

NCNCNC

Insulin levels Ø Ø ≠ (less with glim.) ≠ Glucose-dependentrise

Glucose-dependentrise

NC

Elderly AVOID if >70 years(because Øcreatinineclearance) avoid Cr decrease <70

No special risks Watch for Hypoglycemia

Watch forHypo-glycemia

Ideal Ideal No specialrisks (GI side-effects maybe beneficial)

Preg risk in pts.with PolycysticOvary Syndrome

+ +

32 American Diabetes Association Guidelines and Treatment Algorithms At-A-Glance Appendix

Bibliography1. Byetta Prescribing Information. February 2007. www.byetta.com. Accessed March 2007.

2. Fineman MS, et al: Effect on glycemic control of exenatide (synthetic exendin-4) additive to existing metformin and/or sulfonylureatreatment in patients with type 2 diabetes. Diabetes Care 2003;26(8):2370–7.

3. FDA website. www.fda.gov/medwatch. Accessed June 2005.

4. Buse JB, et al. Effects of exenatide (exendin-4) on glycemic control over 30 weeks in sulfonylurea-treated patients with type 2 diabetes.Diabetes Care 2004;27(11):2628–35.

5. Triplitt CL, et al: Diabetes Mellitus. In Dipro J, et al(eds). Pharmacotherapy: Pathophysiologic approach. 6th ed.New York, NY: McGraw-Hill Companies. 2005;1333–1367

6. The American Association of Clinical Endocrinologists medical guidelines for the management of diabetes mellitus: The AACE system of intensive diabetes self-management 2002 update. Endocrine Practice 2002;8(Suppl. 1):40–65.

7. American Diabetes Association. Diagnosis and classification of diabetes mellitus. Diabetes Care 2005;28(Suppl. 1):S37–S42.

8. American Diabetes Association. Standards of medical care in diabetes. Diabetes Care 2005;28(Suppl. 1):S4–S36.

9. Poulsen MK, et al: The combined effect of triple therapy with rosiglitazone, metformin, and insulin aspart in type 2 diabetic patients.Diabetes Care 2003;26(12):3273–3279.

10. Dailey GE, et al: Glycemic control with glyburide/metformin tablets in combination with rosiglitazone in patients with type 2 diabetes: a randomized, double-blind trial. Am J Med 2004;116:223–229.

11. DeFronzo RA, et al: Effects of exenatide (exendin-4) on glycemic control and weight over 30 weeks in metformin treated patients with type 2 diabetes. Diabetes Care 2005;28(5):1092–1100.

12. Kendall DM, et al: Effects of exenatide (exendin-4) on glycemic control over 30 weeks in patients with type 2 diabetes treated withmetformin and a sulfonylurea. Diabetes Care 2005;28(5):1083–1091.

13. Klein S, et al: Weight management through lifestyle modification for the prevention and management of type 2 diabetes: rational and strategies. Diabetes Care 2004;27(8):2067–2073.

14. Chobanian AV, et al: Seventh Report of the Joint National Committee on prevention, detection, evaluation and treatment of high blood pressure. Hypertension 2003;42:1206–1252.

15. Heine RJ, et al: Exenatide versus insulin glargine in patients with suboptimally controlled type 2 diabetes. Annals of Internal Medicine2005;143:559–569.

16. Nathan DM, et al: Management of hyperglycemia in type 2 diabetes: A consensus algorithm for the initiation and adjustment of therapy – A consensus statement from the American Diabetes Association and the European Association for the Study of Diabetes.Diabetes Care 2006;29(8)1963–1972.

3333Educational (Learning) Critiques National Lipid Association Self-Study Module

American Diabetes AssociationSelf-Assessment ProgramEducational Critiques

IN ORDER TO MAXIMIZE YOUR LEARNING EXPERIENCE IT IS IMPORTANT THAT YOU DO NOT LOOK AT THE EDUCATIONAL CRITIQUES UNTIL AFTER YOU HAVE COMPLETEDMARKING YOUR ANSWERS FOR THE MCQs ON THE ANSWER SHEET.

DO NOT CHANGE THE ANSWERS MARKED ON YOUR ANSWER SHEET WHILE READING THE CRITIQUES. THE INTENT OF ADA-SAP™ IS LEARNING NEW KNOWLEDGE ANDREINFORCING PREVIOUSLY LEARNED KNOWLEDGE. THERE IS NO PASS-FAIL SCORE.

American Diabetes Association Self-Assessment Program Educational (Learning) Critiques34

Educational (Learning) CritiquesItems 1–75

Items 1–6 Answers 1 (A); 2 (B); 3 (C); 4 (C); 5 (D); 6 (C)

The ADA has established three criteria for the diagnosis of diabetes:

1. Symptoms of diabetes and a casual plasma glucose M200 mg/dL (11.1 mmol/L). Casual is defined as any time without regard to time since last meal. The classic symptoms of diabetes include polyuria, polydipsia, andunexplained weight loss, OR

2. Fasting plasma glucose (FPG) M126 mg/dL (7.0 mmol/L). Fasting is defined as no caloric intake for at least 8 hours, OR

3. 2-hour plasma glucose M200 mg/dL (11.1 mmol/L) during an oral glucose tolerance test (OGTT). The test should be performed as described by the World Health Organization, using a glucose load containing the equivalent of 75 g anhydrous glucose dissolved in water.

Impaired fasting glucose (IFG) is defined as FPG of 100–125 mg/dL. Impaired glucose tolerance (IGT) is a 2-hour plasma glucose of 140–199 mg/dL after a 75 g OGTT. Although the OGTT is more sensitive and modestlymore specific than FPG to diagnose diabetes, it is poorly reproducible and rarely performed in practice. Because of ease of use, acceptability to patients, and lower cost, the FPG is the preferred diagnostic test. It should be notedthat the vast majority of people who meet diagnostic criteria for diabetes by OGTT, but not by FPG, will have an A1C value l7.0%. The use of the A1C for the diagnosis of diabetes is not recommended by the ADA at thistime. Pre-diabetes is defined as having either IFG or IGT.

Bibliography1. American Diabetes Association. Standards of medical care in diabetes – 2007. Diabetes Care 2007;30(Supp1).

Item 7 Answer E

According to the ADA guidelines:

1. Testing for diabetes should be considered in all individuals at age 45 years and above, particularly in those with a BMI M25 kg/m2 and, if normal, should be repeated at 3-year intervals.

2. Testing should be considered at a younger age or be carried out more frequently in individuals who areoverweight (BMI M25 kg/m2*) and have additional risk factors:

K Are habitually physically inactive.

K Have a first-degree relative with diabetes.

K Are members of a high-risk ethnic population (e.g., African-American, Latino, Native American,Asian American, Pacific Islander).

K Have delivered a baby weighing L9 lb or have been diagnosed with GDM.

K Are hypertensive (blood pressure, M140/90 mm Hg).

K Have an HDL cholesterol level l35 mg/dL (0.90 mmol/L) and/or a triglyceride level L250 mg/dL (2.82 mmol/L).

K Have polycystic ovary syndrome (PCOS).

K On previous testing, had IGT or IFG.

K Have other clinical conditions associated with insulin resistance (e.g., PCOS or acanthosis nigricans).

K Have a history of vascular disease.

* May not be correct for all ethnic groups.

35Educational (Learning) Critiques American Diabetes Association Self-Assessment Program

This 35-year-old African-American man has at least two CHD risk factors (positive family history of CVD andhypertension) and, therefore, according to the NCEP ATP III guidelines, his LDL goal is l130 mg/dL. JNC7 also recommends drug therapy for a blood pressure of L130/85 mm Hg in patients with multiple risk factors.The ADA does not recommend the routine use of OGTT to diagnose type 2 diabetes but does recommend thetesting of his overweight children for type 2 diabetes. The following is the criteria for testing for type 2 diabetes in children:

K Overweight (BMI L85th percentile for age and sex, weight for height L85th percentile, or weight L120% of ideal for height).

Plus any two of the following risk factors:

K Family history of type 2 diabetes in first- or second-degree relative.

K Race/ethnicity (Native American, African-American, Latino, Asian American, Pacific Islander).

K Signs of insulin resistance or conditions associated with insulin resistance (acanthosis nigricans, hypertension,dyslipidemia, or PCOS).

K Maternal history of diabetes or GDM.

Age of initiation: age 10 years or at onset of puberty, if puberty occurs at a younger age.Frequency: every 2 years.Test: FPG preferred.

Clinical judgment should be used to test for type 2 diabetes in high-risk patients who do not meet these criteria.

Bibliography1. American Diabetes Association. Standards of medical care in diabetes – 2007. Diabetes Care 2007;30(Supp1).

Item 8 Answer C

The ADA states that a graded exercise test with electrocardiogram (ECG) monitoring should be seriouslyconsidered before undertaking aerobic physical activity with intensity exceeding the demands of everyday living(more intense than brisk walking) in previously sedentary patients with diabetes whose 10-year risk of a coronaryevent is likely to be M10%. A diet prescription that focuses on the foods the patient is used to eating should also be implemented.

This patient’s 39-year-old sister does not have a 10-year risk of coronary event M10% (her risk is l5%) and,therefore, a stress test is not recommended. In regard to treatment, the ADA recommends early intervention withmetformin in combination with lifestyle changes (Figure 1). The ADA does not recommend low-carbohydratediets (restricting to total carbohydrate (l130 g/day). The long-term effects of these diets are unknown, andalthough such diets produce short-term weight loss, maintenance of weight loss is similar to that from low-fat dietsand the impact on CVD risk profile is uncertain. Monitoring carbohydrate, whether by carbohydrate counting,exchanges, or experience-based estimation, remains a key strategy in achieving glycemic control. For individualswith diabetes, the use of glycemic index and glycemic load may provide a modest additional benefit for glycemiccontrol over that observed when total carbohydrate is considered alone. Therefore, there is not sufficient evidenceto recommend use of glycemic index or glycemic load for prevention of diabetes, although foods high in fiber areencouraged.

American Diabetes Association Self-Assessment Program Educational (Learning) Critiques36

Figure 1. Algorithm for the metabolic management of type 2 diabetes. Reinforce lifestyle intervention at every visit.

* Check A1C every 3 months until l7% and then at least every 6 months.† Although three oral agents can be used, initiation and intensification of insulin therapy is preferred based on effectiveness and expense.

Sulfonylurea and glitazone are recommended as add-on therapy to metformin if the A1C is L7.0%. Even thoughthis patient has low HDL and glitazone has diabetic dyslipidemic benefits and sulfonylurea lowers glucose rapidly,the primary initial therapy should be metformin due to cost considerations, the absence of hypoglycemia, andevidence of benefit.

Bibliography1. American Diabetes Association. Standards of medical care in diabetes – 2007. Diabetes Care 2007;30(Supp1).

Item 9 Answer A

According to the ADA guidelines:

K Patients with a systolic blood pressure of 130–139 mm Hg or a diastolic blood pressure of 80–89 mm Hgshould be given lifestyle and behavioral therapy alone for a maximum of 3 months and then, if targets are notachieved, in addition, be treated with pharmacological agents that block the rennin-angiotensin system.

K Initial drug therapy for those with a blood pressure L140/90 mm Hg should be with a drug class demonstratedto reduce CVD events in patients with diabetes (ACE inhibitors, angiotensin receptor blockers [ARBs]‚β-blockers, diuretics, and calcium channel blockers).

K All patients with diabetes and hypertension should be treated with a regimen that includes either an ACEinhibitor or an ARB. If one class is not tolerated, the other should be substituted. If needed to achieve bloodpressure targets, a thiazide diuretic should be added.

K In individuals without overt CVD:k The primary goal is an LDL l100 mg/dL (2.6 mmol/L).k For those over the age of 40 years, statin therapy to achieve an LDL reduction of 30–40% regardless

of baseline LDL levels is recommended.k For those under the age of 40 years but at increased risk due to other cardiovascular risk factors who do not

achieve lipid goals with lifestyle modifications alone, the addition of pharmacological therapy is appropriate.

K Other goals include:k Lower triglycerides to l150 mg/dL (1.7 mmol/L) and raise HDL cholesterol to L40 mg/dL (1.0 mmol/L).

In women, an HDL goal 10 mg/dL higher (L50 g/dL) should be considered.

Lifestyle Intervention + Metformin

Diagnosis

Add Sulfonylurea(Least expensive)

Add Glitazone(No hypoglycemia)

Add Basal Insulin(Most effective)

Yes*No A1C M7%

Yes*No A1C M7% Yes*No A1C M7% Yes*No A1C M7%

Intensify Insulin Add Glitazone† Add Basal Insulin Add Sulfonylurea†

Yes*No A1C M7%Yes*No A1C M7%

Add Basal or Intensify Insulin

Intensive Insulin + Metformin +/- Glitazone

37Educational (Learning) Critiques American Diabetes Association Self-Assessment Program

The primary treatment for dyslipidemia is a statin, unless the triglycerides are L500 mg/dL.

Bibliography1. American Diabetes Association. Standards of medical care in diabetes – 2007. Diabetes Care 2007;30(Supp1).

Item 10 Answer E

The ADA recommends all of the following ancillary tests as part of the initial workup of patients with newly-diagnosed type 2 diabetes. Urinary evaluation for microalbuminuria is important to assess early renal damage andpotentially increase the aggressiveness of blood pressure control. Patients with type 2 diabetes should also undergoa dilated comprehensive eye exam by an ophthalmologist or optometrist as well as screening for distal symmetricpolyneuropathy using simple clinical tests (i.e., assess vibratory response in both feet). A comprehensive foot exam is needed to evaluate for potential ulcers or loss of sensation with the appropriate referral for special footwear orpodiatric care.

Bibliography1. American Diabetes Association. Standards of medical care in diabetes – 2007. Diabetes Care 2007;30(Supp1):S4–S41.

2. Nathan DM, et al: Management of hyperglycemia in type 2 diabetes: A consensus algorithm for the initiation and adjustment of therapy:a consensus statement from the American Diabetes Association and the European Association for the Study of Diabetes. Diabetes Care2006;29:1963–1972.

Item 11 Answer A

The ADA website has a risk calculator that can be a very valuable educational tool to assist clinicians to assess riskof diabetes. The website link is www.diabetes.org/diabetesphd. This website is very patient-friendly and can helpeducate regarding the benefits of weight reduction and other factors to reduce the risk of diabetes, heart disease,stroke, and kidney disease. The Diabetes PhD risk calculator can also powerfully demonstrate the benefits of evenmodest weight loss in reducing the risk of type 2 diabetes. In this patient, a 10% weight loss (20 lbs) would reduceher risk of developing type 2 diabetes to less than 10%. Weight loss is the most powerful modifier for the preventionof type 2 diabetes.

In the Diabetes Prevention Program (DPP), the 3234 enrolled subjects were slightly younger (mean age 51 years)and more obese (mean BMI 34 kg/m2) but had nearly identical glucose intolerance compared with subjects in the Finnish study. About 45% of the participants were from minority groups (e.g., African-American, Hispanic),and 20% were M60 years of age. Subjects were randomized to one of three intervention groups, which included the intensive nutrition and exercise counseling (“lifestyle”) group or either of two masked medication treatmentgroups: the biguanide metformin group or the placebo group. The latter interventions were combined withstandard diet and exercise recommendations. After an average follow-up of 2.8 years (range 1.8–4.6 years), a 58%relative reduction in the progression to diabetes was observed in the lifestyle group (absolute incidence 4.8%), anda 31% relative reduction in the progression of diabetes was observed in the metformin group (absolute incidence7.8%) compared with control subjects (absolute incidence 11.0%). On average, 50% of the lifestyle group achievedthe goal of M7% weight reduction, and 74% maintained at least 150 min/week of moderately intense activity.No serious side-effects were seen in any group.

Bibliography1. Diabetes Prevention Research Group: Reduction in the evidence of type 2 diabetes with life-style intervention or metformin.

N Engl J Med 2002;346:393–403.

2. The Diabetes Prevention Program Research Group: The Diabetes Prevention Program: baseline characteristics of the randomizedcohort. Diabetes Care 2000;23:1619–1629.

American Diabetes Association Self-Assessment Program Educational (Learning) Critiques38

Item 12 Answer D

The ADA recently published a consensus statement of the implications for care in patients with pre-diabetes (either impaired fasting glucose and/or impaired glucose tolerance). The ADA Panel recommended, based onclinical trial evidence and cost considerations, that only metformin be considered as drug therapy for individualswith IFG/IGT. In the DPP study, the subsets of the study cohort that had substantially increased benefit frommetformin were those participants l60 years of age and those who had a BMI M35 kg/m2. Therefore, the Panelalso recommends that metformin be limited to such individuals. Since individuals with associated risk factors fordiabetes, e.g., family history in first-degree relatives, elevated triglycerides, low HDL cholesterol, and hypertension,are more likely to progress to diabetes, the presence of one or more of these factors may contribute to the decisionto treat with metformin. In addition, to better target a population likely to benefit from metformin therapy, anunpublished analysis of data from the DPP study suggests that an A1C M6.0% approximately doubles the rate of progression to diabetes in an IFG/IGT population.

A summary of these recommendations in shown in Table 1. Future recommendations may include other medications if they prove to be effective, have a good safety profile, are tolerable, and are of relatively low cost.

Table 1. Treatment recommendation for individuals with IFG, IGT, or both.

* Metformin 850 mg b.i.d.

Bibliography1. Nathan DM, et al: Impaired fasting glucose and impaired glucose tolerance. Diabetes Care 2007;30:753–759.

Item 13 Answer B

The ADA’s treatment algorithm for hyperglycemia strongly encourages the initiation of metformin in conjunctionwith lifestyle changes. Therefore, although this patient has made some progress with weight loss, metformintreatment should be initiated. Sulfonylureas are second-line agents and can effectively lower blood glucose acutelyin symptomatic individuals, but this patient’s hyperglycemia is not severe, and she is asymptomatic. Although she has dyslipidemia, the ADA recommends TZDs as second-line agents but should be considered in patients with low HDL/elevated triglycerides. The hyperglycemic management guidelines were released prior to theapproval of sitagliptin, which is a category B drug for use in women of child-bearing potential, but metformin is still the first-line agent.

Bibliography1. Nathan DM, et al: Management of hyperglycemia in type 2 diabetes: A consensus algorithm for the initiation and adjustment of therapy:

a consensus statement from the American Diabetes Association and the European Association for the Study of Diabetes. Diabetes Care2006;29:1963–1972.

Population Treatment

IFG or IGT Lifestyle modification (i.e., 5–10% weight loss and moderateintensity physical activity p30 min/day)

Individuals with IFG and IGT and any of the following: Lifestyle modification (as above) and/or metformin*K L60 years of age.K BMI M35 kg/m2.K Family history of diabetes in first-degree relatives.K Elevated triglycerides.K Reduced HDL-C.K Hypertension.K A1C L6.0%.

39Educational (Learning) Critiques American Diabetes Association Self-Assessment Program

Items 14–18 Answers 14 (D); 15 (B); 16 (A); 17 (E); 18 (C)

Thiazolidinediones are contraindicated in patients with NYHA Class III/IV cardiac failure because of concernsthat they may precipitate pulmonary edema, but there is no evidence that they are harmful in Class I heart failure.Rosiglitazone has recently been implicated as increasing the incidence of myocardial infarction and a borderlinesignificant increase in death in a meta-analysis. Thiazolidinediones also cause a modest reduction in the hematocritprobably by a hemdilution effect. Gemfibrozil, but not fenofibrate, inhibits glucuronidation of statins. This is apathway that had not been recognized earlier as being important in statin catabolism. As a consequence, statinlevels have been demonstrated to be increased in subjects receiving gemfibrozil. Fenofibrate is extensively metabo-lized in the kidney, and therefore should be dose-adjusted in patients with moderate-to-severe renal impairment.Gemfibrozil requires dose adjustment only in patients with severe renal impairment. In the clinical scenariodescribed in Item 17 the thiazolidinediones and fibrate are not contraindicated since the liver enzymes are less than twice elevated, and these drugs may improve fatty liver changes.

Bibliography1. Nesto RW, et al: American Heart Association; American Diabetes Association. Thiazolidinedione use, fluid retention, and congestive

heart failure: a consensus statement from the American Heart Association and American Diabetes Association. Circulation2003;108(23):2941–8.

2. Lebovitz HE. Differentiating members of the thiazolidinedione class: a focus on safety. Diabetes Metab Res Rev 2002;18(Suppl 2):S23–9.

3. Prueksaritanont T, et al: Effects of fibrates on metabolism of statins in human hepatocytes. Drug Metab Dispos 2002;30(11):1280–7.

4. Gajdos M, et al: Fibrates and renal function. Clin Nephrol 2003;60(1):65–6.

5. On-Line: N Engl J Med 5/20/07.

Items 19–23 Answers 19 (A); 20 (A); 21 (C); 22 (D); 23 (E)

The importance of adipocytokines to the genesis of insulin resistance and atherosclerosis is assuming increasingimportance. Among these, adiponectin has been shown to both enhance insulin sensitivity and inhibit monocyteadhesion and infiltration of the vascular wall. By contrast, free fatty acids cause insulin resistance in part by inter-fering with hepatic insulin degradation causing hyperinsulinemia which leads to down-regulation of insulinreceptors and insulin resistance. TNF-α increases insulin resistance by interfering with insulin-signaling pathways.PAI-1 localizes in plaque where it may promote coagulation.

Bibliography1. Molecular mechanisms of insulin resistance and the role of the adipocyte: Int J Obes Relat Metab Disord 2000;24(Suppl)4:S23–7.

2. Hoffstedt J, et al: Adipose tissue adiponectin production and adiponectin serum concentration in human obesity and insulinresistance.J Clin Endocrinol Metab 2004;89(3):1391–6.

3. Hennes MM, et al: Effects of free fatty acids and glucose on splanchnic insulin dynamics. Diabetes 1997;46(1):57–62.

Item 24 Answer B

The biguanides, represented by metformin in the United States, are effective oral hypoglycemic agents. They arealso effective when added to other oral hypoglycemic agents, such as thiazolidinediones or sulfonylureas, or whenadded to insulin. They are usually dosed with meals. Use of biguanides has been associated with lactic acidosisrarely with no obvious cause (50% death rate if it does occur) and especially in patients with renal insufficiency,significant liver disease, or compromised LV function (90% death rates).

Bibliography1. Mudaliar S, et al: The oral antidiabetic agents. In: Ellenberg and Rifkin’s Diabetes Mellitus, 6th Ed. Porte D, et al (eds).

McGraw-Hill, 2003;531–564.

American Diabetes Association Self-Assessment Program Educational (Learning) Critiques40

Item 25 Answer B

Insulin resistance is associated with increased risk for cardiovascular disease. There have been multiple mecha-nisms suggested for this association. However, recently there has been increasing support for the pro-inflammatorystate that accompanies insulin resistance as being pathophysiologically important for accelerated large vesseldisease. Patients with insulin resistance sometimes have elevated homocysteine levels. However, the relationshipbetween homocysteine and the accelerated vascular disease of insulin resistance is less strong (therapy with folicacid decreases homocysteine levels but does not reduce CV events). Patients with insulin resistance have increasedplasminogen activator inhibitor levels and, therefore, a pro-thrombotic state. While patients with insulin resistancealso have elevated insulin levels, the association between insulin levels and vascular disease is most likely related to insulin resistance and not to direct toxic effect of insulin on the vasculature.

Bibliography1. Haffner S, et al: Insulin-resistant prediabetic subjects have more atherogenic risk factors than insulin-sensitive prediabetic subjects.

Implications for preventing coronary heart disease during the prediabetic state. Circulation 2000;101:975–980.

2. Hunt K, et al: Elevated carotid artery intima-media thickness levels in individuals who subsequently develop Type 2 diabetes.Arterioscler Thromb Vasc Biol 2003;23:1845–1850.

3. Soinio M, et al: Elevated plasma homocysteine level is an independent predictor of coronary heart disease events in patients with Type 2 diabetes mellitus. Ann Intern Med 2004;140:94–100.

Item 26 Answer D

The ability to monitor blood glucose at home has revolutionized management of patients with diabetes. Bloodglucose levels are of value to physicians, and can be obtained either several times per week or several times per day,depending on the clinical circumstance. While patients on insulin require more frequent monitoring patients withtype 2 diabetes on oral hypoglycemic therapy will also benefit from home monitoring. For patients on insulintherapy, measuring the fasting blood glucose is the most reliable indicator for adjustment of the basal insulin dose.In the most intensively controlled patients, it is useful to measure blood glucose before meals and at bedtime.

Bibliography1. American Diabetes Association: Clinical Practice Recommendations 2004. Standards of medical care in diabetes.

Diabetes Care 27(Suppl. 1):2004;S15–S35.

41Educational (Learning) Critiques American Diabetes Association Self-Assessment Program

Items 27–31 Answers 27 (A); 28 (D); 29 (C); 30 (B); 31 (E)

There are now several classes of oral hypoglycemic agents available. Because these major classes have comple-mentary mechanisms of action, they are often useful as combination therapy in individual patients. Sulfonylureaswork primarily by increasing insulin secretion from β-cells. Thiazolidinediones are insulin sensitizers that increaseglucose disposal by increasing its uptake into muscle. The biguanides have a favorable effect on fasting blood glucoseby suppressing glucose production in the liver. The alpha-glucosidase inhibitors are not as efficacious for reducingoverall glycemia as sulfonylureas, thiazolidinediones or biguanides; however, they are useful for reducing post-prandial blood glucose spikes, as they delay glucose uptake in the intestine and, thereby, inhibit glucose absorption.

Figure 2. Mechanism of action of sitagliptin.

The incretin hormones GLP-I and GIP are released by the intestine throughout the day, and levels are increased inresponse to a meal. The incretins are part of an endogenous system involved in the physiologic regulation of glucosehomeostasis.

When blood glucose concentrations are normal or elevated, GLP-I and GIP increase insulin synthesis and releasefrom pancreatic β-cells by intracellular signaling pathways involving cyclic AMP. With higher insulin levels, tissueglucose uptake is enhanced.

In addition, GLP-I lowers glucagon secretion from pancreatic α-cells.

K Decreased glucagon levels, along with higher insulin levels, lead to reduced hepatic glucose production and are associated with a decrease in blood glucose levels in the fasting and post-prandial states.

K The beta-cell effects of GLP-I and GIP are glucose-dependent, but only the effect of GLP-1 on the alpha cell is glucose-dependent. The glucose-dependency effect of GLP-1 on alpha- and beta-cells translate to a benefit of rare hypoglycemia with incretin mimetics.

The activity of GLP-I and GIP is limited by the DPP-4 enzyme, which rapidly inactivates incretin hormones.

Concentrations of the active intact hormones are increased by sitagliptin, thereby increasing and prolonging theaction of these hormones. (Figure 2).

Bibliography1. Mudaliar S, et al: The oral antidiabetic agents. In: Ellenberg and Rifkin’s Diabetes Mellitus, 6th Ed. Porte D, et al (eds). McGraw-Hill,

2003;531–564.

Ingestion of food

GI tractPancreasBeta cellsAlpha cells

Glucosedependent

InactiveGLP-1

InactiveGIP

Release ofactive incretinsGLP-1 and GIP

DPP-4enzyme

Sitagliptin(DPP-4 inhibitor)

QGlucagon (GLP-1)

GlucosedependentqInsulin

(GLP-1and GIP) qGlucose

uptake byperipheral

tissueQBlood glucose

in fasting and postprandial states

QHepaticglucose

production

American Diabetes Association Self-Assessment Program Educational (Learning) Critiques42

Items 32–35 Answers 32 (B); 33 (C); 34 (A); 35 (D)

The oral hypoglycemic agents are generally safe; however, each major class is associated with specific serious side-effects. Sulfonylureas, because they increase insulin secretion, are the class of oral hypoglycemic agents most closelyrelated to production of symptomatic hypoglycemia. The thiazolidinediones may increase fluid retention andworsen peripheral edema. Their use in patients with symptomatic congestive heart failure is, therefore, not recom-mended. Biguanides most often give rise to gastrointestinal side-effects including mild nausea and some diarrhea,but this can be minimized by titrating to a maximal effective dose of 2 g/day over 2 weeks. These side-effects aregenerally self-limited. However, in patients with renal insufficiency or impaired cardiac function, biguanides havebeen rarely associated with lactic acidosis (see Critique for Item 24). Alpha-glucosidase inhibitors decrease thedigestion of carbohydrates which are converted into gas and can cause bloating.

Bibliography1. Mudaliar S, et al: The oral antidiabetic agents. In: Ellenberg and Rifkin’s Diabetes Mellitus, 6th Ed. Porte D, et al (eds).

McGraw-Hill, 2003;531–564.

Item 36 Answer D

All of the options in this item have been shown to be beneficial as each can lead to weight loss. A reduction inweight of as little as 4 kg has been shown to improve hyperglycemia. Whereas dietary and physical activity adjust-ments have been universally recommended and remarkably useful in some patients, weight loss following bariatricsurgery has been shown to be consistently effective in virtually eliminating diabetes if a mean weight loss of 20 kg or more can be sustained. Although not appropriate for all patients, in properly-selected obese patients, bariatricsurgery can be an effective option. The results of recent studies have shown that bariatric surgery decreasedmortality rate in selected obese patients with type 2 diabetes. Studies of weight loss medications, such as orlistatwhich reduces fat absorption and sibutramine which causes a modest decrease in appetite have demonstrated highdrop-out rates, significant side-effects, and cannot be recommended at this time for the treatment of diabetes.

Bibliography1. Sjostrom L, et al: Swedish Obese Subjects Study Scientific Group. Lifestyle, diabetes, and cardiovascular risk factors 10 years after

bariatric surgery. N Engl J Med 2004;351:2683–2693.

2. Nathan DM, et al: Management of hyperglycemia in type 2 diabetes: A consensus statement from the American Diabetes Associationand the European Association for the Study of Diabetes. Diabetes Care 2006;29:1963–1972.

Item 37 Answer E

Exercise and weight loss have a beneficial effect on diabetes control but also on associated cardiovascular risk factors, such as blood pressure and lipids. There is a psychological boost as well in exercising regularly and seeing this improve your glycemic control and other risk factors. While lower weight and improved diabeticcontrol may slow the progression of neuropathy in the long-term, in the short-term the newly-exercising patientwith neuropathy is often at increased risk of foot trauma or ulceration. One should caution all patients withdiabetes, especially those with neuropathy, to look carefully at their feet everyday, particularly after exercise.It is also important to have properly-fitting shoes and other equipment to lessen the possibility of injury.

Bibliography1. Nathan DM, et al: Management of hyperglycemia in type 2 diabetes: A consensus statement from the American Diabetes Association

and the European Association for the Study of Diabetes. Diabetes Care 2006;29:1963–1972.

2. Diabetes Prevention Program Research Group: Impact of intensive lifestyle and metformin therapy on cardiovascular disease risk factorsin the Diabetes Prevention Program. Diabetes Care 2005;28:888–894.

43Educational (Learning) Critiques American Diabetes Association Self-Assessment Program

Item 38 Answer C

Metformin is recommended as first line of treatment in most patients with newly-diagnosed diabetes that requiremedication for all of the reasons mentioned in this item, except for option (C). While metformin is generally well-tolerated it may have gastrointestinal side-effects, such as diarrhea or nausea, especially at higher doses.More worrisome is the potentially fatal possibility of lactic acidosis which is actually very rare (l1 case per 100,000treated patients). The risk of lactic acidosis can be reduced by avoiding metformin in patients with renal insuffi-ciency (creatinine L1.4 mg/dL in women, L1.5 mg/dL in men), in association with the use of iodinated contrast or with any condition that predisposes the patient to hypotension or hypoxia.

Bibliography1. Nathan DM, et al: Management of hyperglycemia in type 2 diabetes: A consensus statement from the American Diabetes Association

and the European Association for the Study of Diabetes. Diabetes Care 2006;29:1963–1972.

2. Salpeter S, et al: Risk of fatal and nonfatal lactic acidosis with metformin use in type 2 diabetes mellitus. Cochrane Database Syst Rev2006;CD002967.

Item 39 Answer C

The sulfonylurea medications increase insulin secretion which lowers blood glucose levels. This can causeincreased hunger, hypoglycemia and weight gain. It is common for a patient to gain about 2 kg when started on a sulfonylurea, so they must be used with caution in those that are already overweight. Caution should also beadvised in the elderly that are more prone to hypoglycemia and drug interactions. Urinary tract infections (UTIs)are common in patients with diabetes and sulfa drugs are commonly prescribed for UTIs. However, in one study of over 900 elderly patients admitted to the hospital for hypoglycemia related to a taking a sulfonylurea, comparedwith matched controls the patients with hypoglycemia were more than 6 times as likely to have been treated with a sulfa antibiotic in the previous week. There is no logic to prescribe a sulfonylurea and glinide together as both act on the same receptor on the beta-cell. Like almost any medication, the sulfonylureas can cause a rash.

However, a clear consistent adverse effect on cardiovascular risk has not been established. While the UniversityGroup Diabetes Program suggested an increase in mortality when sulfonylureas are used in patients with type 2diabetes, this was not confirmed by the UK Prospective Diabetes Study.

Bibliography1. Juurlink DN, et al: Drug-drug interactions among elderly patients hospitalized for drug toxicity. JAMA 2003 Apr 2;289(13):1652–8.

PMID: 12672733.

2. Klimt CR, et al: The University Group Diabetes Program: a study of the effect of hypoglycemic agents on vascular complications in patients with adult-onset diabetes. I. Design, methods and baseline characteristics. II. Mortality results.Diabetes 1970;19(Suppl. 2):747–830.

3. UK Prospective Diabetes Study (UKPDS) Group: Intensive blood glucose control with sulphonylureas or insulin compared with conventional treatment and risk of complication in patients with type 2 diabetes (UKPDS 33). Lancet 1998;352:837–853.

American Diabetes Association Self-Assessment Program Educational (Learning) Critiques44

Item 40 Answer C

The only significant drug-drug interaction with the glinides (e.g., repaglinide, nateglinide) concerns gemfibrozil,which can lead to hypoglycemia by reducing the metabolism of either glinide drug by the liver. Therefore, whenhypertriglyceridemia and diabetes coexist, which often occurs, caution must be exercised in choosing therapy.A method other than gemfibrozil, such as control of diabetes, dietary measures, fenofibrate or omega-3 fatty acidsshould be used if triglyceride-lowering is needed.

Bibliography1. Hasslacher C. Multinational Repaglinide Renal Study Group. Safety and efficacy of repaglinide in type 2 diabetic patients with and

without impaired renal function. Diabetes Care 2003 Mar;26(3):886–91.

2. Blickle JF. Meglitinide analogues: a review of clinical data focused on recent trials. Diabetes Metab 2006 Apr;32(2):113–20.

Item 41 Answer A

Metformin is associated with an expected decrease in A1C level of 1.5%. The only biguanide presently marketed in the United States, metformin has been called an “insulin sensitizer” because it lowers blood glucose levelswithout directly increasing insulin production or secretion by the pancreas. Its main mechanism however is to reduce gluconeogenesis in the liver, thereby reducing glucose production. It likely exerts this effect through alterations in AMP-kinase. Reductions in insulin resistance in the peripheral tissues have been observed but theresults are inconsistent.

Bibliography1. Inzucchi SE, et al: Efficacy and metabolic effects of metformin and troglitazone in type II diabetes mellitus. N Engl J Med

1998;338:867–872.

2. Hundal RS, et al: Mechanism by which metformin reduces glucose production in type 2 diabetes. Diabetes 2000;49:2063–2069.

3. Yu JG, et al: A comparison of troglitazone and metformin on insulin requirements in euglycemic intensively insulin-treated type 2diabetic patients. Diabetes 1999;48:2414–242.

Item 42 Answer D

Glitazones are associated with an expected decrease in A1C level of 0.5–1.4%. Pioglitazone and rosiglitazone arethe two thiazolidinediones (TZDs) that are currently available. The primary mechanism of action of the TZDs is toenhance insulin sensitivity by improving the utilization of glucose by the peripheral adipocytes and skeletal muscle.TZDs create this effect by stimulating a nuclear transcription factor called PPAR-γ. These receptors impact lipidand carbohydrate metabolism by activating many (L1000), not just several, genes in various tissues. PPAR-γ ismost highly expressed in adipocytes and TZDs and are thought to act within fat cells to effect the production offree fatty acids, adiponectin and TNF-α, all of which impact insulin sensitivity. Some have described a reduction inhepatic glucose production but not to the extent of metformin.

Bibliography1. Frias JP, et al: Metabolic effects of troglitazone therapy in type 2 diabetic, obese, and lean normal subjects. Diabetes Care 2000;23: 64–69.

2. Nolan JJ, et al: Improvement in glucose tolerance and insulin resistance in obese subjects treated with troglitazone. N Engl J Med1994;331:1188–1193.

Item 43 Answer C

Alpha-glucosidase inhibitors (AGIs) are associated with an expected decrease in A1C level of 0.5–0.8%.This drug class includes acarbose and miglitol. AGIs work by delaying the absorption of carbohydrates in the gut. This has the most significant effect on the post-prandial blood glucose level so these medications are takenwith meals. The AGIs directly inhibit an enzyme that breaks down complex carbohydrates on the brush border ofthe enterocyte. Unfortunately, the mechanism that creates their efficacy also yields their most significant side-effect(intestinal bloating and gas). This is because the carbohydrates that are not absorbed are acted upon by the flora of the gut to produce gas.

45Educational (Learning) Critiques American Diabetes Association Self-Assessment Program

Bibliography1. Lebowitz HE. α-glucosidase inhibitors as agents in the treatment of diabetes. Diabetes Rev 1998;6:132–145.

2. Hanefeld M, et al: Therapeutic potentials of acarbose as first-line drug in NIDDM insufficiently treated with diet alone. Diabetes Care1991;14 :732–737.

3. Hotta N, et al: Long-term effect of acarbose on glycaemic control in non-insulin-dependent diabetes mellitus: a placebo-controlleddouble-blind study. Diabet Med 1993;10:134–138.

Item 44 Answer B

Glinides (also known as meglitinides) are associated with an expected decrease in A1C level of 1–1.5%. Twoglinides are available in the US at this time. Repaglinide is somewhat more efficacious, reducing A1C levels by1.5%. Nateglinide when used as monotherapy reduces A1C by about 1%. Each of these works by acting upon thesame receptor as the sulfonylureas although at a different site. This causes a sequence of events that includesclosure of the ATP-dependent K channels which stimulate insulin secretion from the pancreas. These medicationsare taken just prior to meals and act primarily on the post-prandial glucose level.

Bibliography1. Scarsi M,et.al: Sulfonylureas and Glinides Exhibit PPAR-γ Activity: A Combined Virtual Screening and Biological Assay Approach.

Mol Pharmacol 2006 Nov 3.

2. Nathan DM, et al: Management of hyperglycemia in type 2 diabetes: A consensus statement from the American Diabetes Associationand the European Association for the Study of Diabetes. Diabetes Care 2006;29:1963–1972.

Item 45 Answer E

Sitagliptin is associated with an expected decrease in A1C level of 0.5–1.5%. Sitagliptin is a dipeptidyl peptidase-4(DPP-IV) inhibitor. This acts upon the incretin system, which is a group of hormones active in the gut. One ofthese hormones is GLP-I. DPP-IV inhibitors reduce the breakdown of many peptides, including GLP-I, therebyincreasing their activity. The main effect is to improve glucose-dependent insulin secretion and a glucose-dependent suppression of inappropriate glucagon secretion. In addition, it results in an increased endogenousGLP-1 effect of 2–4 times normal, which does not sufficiently suppress appetite or slow gastric emptying enough to decrease weight. Though it is weight-neutral, it has no increased GI side-effects compared to placebo.

Bibliography1. Nathan DM, et al: Management of hyperglycemia in type 2 diabetes: A consensus statement from the American Diabetes Association

and the European Association for the Study of Diabetes. Diabetes Care 2006;29:1963–1972.

2. Aschner P, et al: Effect of the dipeptidyl peptidase-4 inhibitor sitagliptin as monotherapy on glycemic control in patients with type 2diabetes. Diabetes Care 2006 Dec;29(12):2632–7.

Item 46 Answer C

The only antihypertensive agents proven to prevent progression to microalbuminuria are the angiotensin-converting enzyme inhibitors (ACE-I). A recent review looked at the question of preventing nephropathy inpatients with type 1 or type 2 diabetes who had normal kidney function with or without hypertension. The absenceof nephropathy was defined as albumin excretion less than 30 mg/day on three serial measurements. Most of thetrials reviewed involved patients with type 2 diabetes with hypertension.

ACE-Is were the only agents that clearly showed evidence of preventing progression to microalbuminuria comparedto placebo or calcium channel blockers. The number of patients with diabetes one needed to treat to preventprogression of one patient to microalbuminuria was 25. ACE-Is did not reduce progression to end-stage renaldisease or all-cause mortality. There were not enough studies to determine if normotensive patients received anybenefit.

American Diabetes Association Self-Assessment Program Educational (Learning) Critiques46

Angiotensin receptor blockers (ARBs) have also been shown to slow the progression of nephropathy associatedwith diabetes. The summary of a very recent review of the effect of ACE-Is and ARBs is worth quoting in full.“Fifty studies (13215 patients) were identified. Thirty eight compared ACE-I with placebo, five compared ARBswith placebo and seven compared ACE-Is and ARBs directly. There was no significant difference in the risk of all-cause mortality for ACE-I versus placebo (RR 0.91, 95% CI 0.71–1.17) and ARBs versus placebo (RR 0.99,95% CI 0.85–1.17). A subgroup analysis of studies using full-dose ACE-Is versus studies using half or less thanhalf the maximum tolerable dose of ACE-Is showed a significant reduction in the risk of all-cause mortality with the use of full-dose ACE-I (RR 0.78, 95% CI 0.61–0.98). Baseline mortality rates were similar in the ACE-I andARB studies. The effects of ACE-Is and ARBs on renal outcomes (ESKD, doubling of creatinine, prevention ofprogression of micro- to macroalbuminuria, remission of micro- to normoalbuminuria) were similarly beneficial.Reliable estimates of effect of ACE-Is versus ARBs could not be obtained from the three studies in which they were compared directly because of their small sample size.”

Bibliography 1. Strippoli GFM, et al: Antihypertensive agents for preventing diabetic kidney disease. The Cochrane Database of Systematic Reviews

2006 Issue 4.

2. Strippoli GFM, et al: Angiotensin converting enzyme inhibitors and angiotensin II receptor antagonists for preventing the progression of diabetic kidney disease. The Cochrane Database of Systematic Reviews 2006 Issue 4.

Item 47 Answer B

While nearly all of the published studies regarding the use of statins and prevention of cardiovascular disease eventrates have shown a reduction, the Prospective Study of Pravastatin in the Elderly at Risk (PROSPER) trial found a trend toward harm regarding overall mortality. This trend did not reach the level of statistical significance.

In the PROSPER study 5804 people aged 70–82 years were given pravastatin (40 mg/day; n = 2891) or placebo (n = 2913). Follow-up was 3.2 years on average and our primary endpoint was a composite of coronary death,non-fatal myocardial infarction, and fatal or non-fatal stroke. Pravastatin lowered LDL cholesterol concentrationsby 34% and reduced the incidence of the primary endpoints (hazard ratio 0.85, 95% CI 0.74–0.97, P = 0.014).Coronary heart disease death and non-fatal myocardial infarction risk were also reduced (0.81, 0.69–0.94,P = 0.006). Stroke risk was unaffected (1.03, 0.81–1.31, P = 0.8), but the hazard ratio for transient ischaemic attack was 0.75 (0.55–1.00, P = 0.051). New cancer diagnoses were more frequent on pravastatin than on placebo(1.25, 1.04–1.51, P = 0.020). However, a meta-analysis of five studies found no association between statin use and the risk of non-fatal and fatal cancers. All the other options in this item are correct.

Bibliography 1. Shepherd J, et al: Pravastatin in elderly individuals at risk of vascular disease (PROSPER): A randomised controlled trial.

Lancet 2002;360:1623–30.

Item 48 Answer D

Alpha-glucosidase inhibitors (AGIs), such as acarbose, work by delaying the intestinal absorption of glucose. Theyact primarily on post-prandial blood glucose levels and are unlikely to cause hypoglycemia. They generally have tobe dosed with each meal and the dose of acarbose is 50 mg t.i.d. The advantage of this class of drugs is they areunlikely to cause any systemic effects and are useful in patients with kidney or liver insufficiency where other drugsmay be contraindicated. Unfortunately, their use is limited by side-effects primarily gastrointestinal in that theundigested sugars are acted upon by intestinal bacteria to produce gas. Therefore patients with low tolerance forside-effects would not be good candidates for this class of medications.

The AGIs are less potent than other classes of medications in reducing A1C from 0.5–0.8% so they are not the firstchoice for monotherapy if a large drop in A1C is needed. Thus a patient poorly-controlled with a low tolerance formedication would be the least suitable candidate for this class of medications. A recent review noted that there is noevidence that AGIs reduce morbidity or mortality.

47Educational (Learning) Critiques American Diabetes Association Self-Assessment Program

Bibliography 1. Nathan DM, et al: Management of hyperglycemia in type 2 diabetes: A consensus statement from the American Diabetes Association

and the European Association for the Study of Diabetes Mellitus, Diabetes Care 2006;29:1963–1972.

2. Van de Laar FA, et al: Alpha-glucosidase inhibitors for type 2 diabetes. The Cochrane Database of Systematic Reviews 2006 Issue 4.

Item 49 Answer C

Each of the options listed (A, B, D, E), except option (C), is associated with significant insulin deficiency. Whetherthe fasting blood glucose level is high (L250 mg/dL), the A1C is L10%, or the patient is symptomatic withpolyuria and polydypsia, the quickest way to stabilize the patient and reduce their blood glucose is by startinginsulin. When ketonuria is present it also indicates that the peripheral tissues are unable to utilize glucose as a fueland must resort to lipolysis. Often insulin can be started, quickly titrated to optimum levels and oral agents can beadded. As the blood glucose level corrects, eventually the insulin can often be withdrawn. The initiation of insulinoften is associated with a weight gain of 2–4 kg as the glycosuria is arrested and these calories are no longer lost.However, the specific body mass index should not be a criterion for deciding on the initiation of insulin therapy.

Bibliography 1. Nathan DM, et al: Management of hyperglycemia in type 2 diabetes: A consensus statement from the American Diabetes Association

and the European Association for the Study of Diabetes Mellitus. Diabetes Care 2006;29:1963–1972.

Item 50 Answer A

The American Diabetes Association (ADA) and the European Association for the Study of Diabetes (EASD)recently issued a consensus statement regarding the management of hyperglycemia in patients with type 2 diabetes.In this statement the goals and principles of therapy were outlined and the various options for treatment discussed.There are many factors to consider when lifestyle modifications are inadequate to control the blood glucose level andthe decision to start a patient on medication has been made. Considerations include the goals of the patient, the antici-pated short- and long-term benefits of therapy, and the short- and long-term risks, possible side-effects, expectedadherence to therapy, cost of the medication, insurance coverage and non-glycemic effects of the medication.

The patient described in this item is a common one seen in clinical practice. She is overweight, hypertensive andrequires a significant reduction in her A1C level to be at her goal. Metformin is an excellent initial choice for herstarting medication as it is generally well tolerated, has a low incidence of hypoglycemia, and generally does not leadto additional weight gain. It is also available in a generic formulation, and is relatively effective in controlling bloodglucose.

A sulfonylurea, while inexpensive, is more likely to cause hypoglycemia and additional weight gain. A glitazone hasthe advantage of possibly improving the patient’s lipid profile, but may cause water retention or weight gain and is much more expensive than either metformin or a sulfonylurea. Inhaled insulin is relatively new to the US market,is expensive and would not be considered as a first line medication in most patients. Sitagliptin, a DPP-IV inhibitorwhich acts upon the incretin system to increase insulin release, is also new to the market, expensive and too novel torecommend as starting therapy in most patients. For these reasons and others, the consensus panel recommendedmetformin as the initial choice of drug in most situations. “Metformin is recommended as the initial pharmacologictherapy, in the absence of specific contraindications, for its effect on glycemia, absence of weight gain or hypo-glycemia, generally low level of side-effects, high level of acceptance, and relatively low cost.” Table 2 summarizesthe effects of multiple treatment options in diabetes.

American Diabetes Association Self-Assessment Program Educational (Learning) Critiques48

Table 2. Summary of anti-diabetic interventions as monotherapy.

* Severe hypoglycemia is relatively infrequent with sulfonylurea therapy. The longer-acting agents (e.g., chlorpropamide, glyburide [glibenclamide], and sustained-release glipizide) are more likely to cause hypoglycemia than glipizide, glimepiride, and gliclazide. Repaglinide is more effective at lowering A1C than nateglinide.

Bibliography 1. Nathan DM, et al: Management of hyperglycemia in type 2 diabetes: A consensus statement from the American Diabetes Association

and the European Association for the Study of Diabetes Mellitus. Diabetes Care 2006;29:1963–1972.

Item 51 Answer A

The consensus statement of the ADA and EASD recommends an algorithm for the introduction of insulin if that is deemed necessary (Figure 3). After allowing for patient characteristics, lifestyle, and meal schedules,it is recommended to start with 10 units or 0.2 units/kg of bedtime intermediate-acting insulin or a similar dose of long-acting insulin taken at bedtime or in the morning. A single dose of insulin is likely to have better complianceand by starting low and titrating up, the chances of hypoglycemia are minimized. Blood glucose levels should bemonitored at least daily and the dose adjusted frequently (typically by 2 units every 3 days) until a desired fastinglevel of blood glucose is obtained (70–130 mg/dL). If the fasting glucose is L180 mg/dL, a faster titration may be initiated by increasing the dose by 4 units every 3 days.

Expected decreaseInterventions in A1C (%) Advantages Disadvantages

Step 1: Initial Therapy

Lifestyle to decrease weight andincrease activity 1–2 Low cost, many benefits Fails for most in first year

Metformin 1.5 Weight neutral, inexpensive GI side-effects, rare lactic acidosis

Step 2: Additional Therapy

Insulin 1.5–2.5 No dose limit,inexpensive, Injections, monitoring,improved lipid profile hypoglycemia, weight gain.

Sulfonylureas 1.5 Inexpensive Weight gain, hypoglycemia*

TZDs 0.5–1.4 Improved lipid profile Fluid retention, weight gain, expensive

Other Drugs

α-Glucosidase inhibitors 0.5–0.8 Weight neutral Frequent GI side-effects,t.i.d. dosing, expensive.

Exenatide 0.5–1.0 Weight loss Injections, frequent GI side-effects, expensive, little experience

Glinides 1–1.5† Short duration t.i.d. dosing, expensive

Pramlintide 0.5–1.0 Weight loss Injections, t.i.d. dosing, frequent GI side-effects, expensive, little experience

49Educational (Learning) Critiques American Diabetes Association Self-Assessment Program

Figure 3. Algorithm for starting insulin.

Initiation and adjustment of insulin regimens. Insulin regimens should be designed taking lifestyle and meal schedule into account. The algorithm can only provide basic guidelines for initiation and adjustment of insulin. Premixed insulins are not recommended during adjustment of doses; however, they can be usedconveniently, usually before breakfast and/or dinner if proportion of rapid- and intermediate-acting insulins is similar to the fixed proportions available.

bg = blood glucose.

Bibliography 1. Nathan DM, et al: Management of hyperglycemia in type 2 diabetes: A consensus statement from the American Diabetes Association

and the European Association for the Study of Diabetes Mellitus. Diabetes Care 2006;29:1963–1972.

Start with bedtime intermediate-acting insulinor bedtime or morning long-acting insulin. Can initiate with 10 units or 0.2 units per kg

Check fast ing g lucose(f ing ersti ck) us uall y d aily andincr ease dose, ty pically by2 un itsev ery 3d ays unt ilf ast ing lev els

arei n targ etrang e (70– 130 mg/d L o r 3 .89 –7.2 2 m mol/ L); cani ncr eas e dosein lar geri ncrem ents , e. g.b y 4 units ev ery

3d ays, iffasting glu cose L180 mg/ dL( L10m mol/ L)

A1C M7%afte r 2– 3 m ont hs ?

Ifh ypogl ycemi a occur s,orfa sti ng gluc osele vel L70 mg/dL

(3.8 9 m mol/L), re duc e be dti medo se byM4un its , or1 0 %if dos e L60uni ts

Iffa sti ngbg int arget ra nge( 70– 130 mg/ dL) or 3.8 9–7. 22 m mol /L), ch eck bg pre-l unch, -din ner ,

and -bed, dep endi ng onb g r esu lts , ad d s econ d in ject ion; can us ually beg in with ~4u nit s

and ad just by 2u nit s e ver y 3 day s u nti l bg in ran ge

Contin ue re gimen;check A1C

every 3mon ths

Pre-l unc h bgout of ran ge :add rap id-a cti ng

insulinatbr eak fast

Pre-di nner bg ou t o f rang e :add NPH ins uli n

atbr eakf ast or rap id-ac tin g at lu nch

A1C M7%af ter 3

Rech eck pre-meal bg lev els an d ifoutof ran ge,ma y n ee d t o ad d ano the r i n ject ion;

if A1Cc ont inu es to beo ut ofra nge, check2 -h p ost -pran dial lev els an d ad just

pre-p rand ial rapi d-a cti ngi nsul in

Pre-be d bg out ofrang e :add ra pid- act ing

ins ulin at dinn er

NO YES

YES

NO

American Diabetes Association Self-Assessment Program Educational (Learning) Critiques50

Item 52 Answer B

Since secretogogues, such as sulfonylureas and glinides, increase insulin secretion, they should be discontinued ifshort-acting insulin is added to the patient’s regimen. Otherwise, there is a high risk of hypoglycemia and it is moredifficult to titrate the correct dose of the short-acting insulin. It is also expensive and unnecessary to continue bothtypes of therapy. The basal insulin should be continued however in that the short-acting insulin will primarily affectthe post-prandial blood glucose level and be inadequate to control the blood glucose level during fasting periods,such as sleep. ACE-Is, while useful in preserving renal function, have no direct bearing on the initiation of short-acting insulins. Carbohydrate intake should remain constant; otherwise the risk of hypoglycemia will be increased.

Bibliography 1. Nathan DM, et al.: Management of hyperglycemia in type 2 diabetes: A consensus statement from the American Diabetes Association

and the European Association for the Study of Diabetes Mellitus. Diabetes Care 2006;29:1963–1972.

Item 53 Answer D

All the listed options (A, B, C, E) except option (D) (increasing the metformin to 1000 mg t.i.d.) are acceptablealternatives. The maximum recommended daily dose of metformin is 2550 mg. The maximum recommended doseof the extended-release form of metformin is 2000 mg. In the recent consensus panel of the ADA and EASD, therewas no clear preferred choice for the second agent to add in this situation. A glitazone, sulfonylurea or a single doseof basal or intermediate-acting insulin are all acceptable options. The choice of insulin is favored if the A1C isL8.5% or the patient is symptomatic from the elevated blood glucose. Other factors to consider include the cost of the medication and patient preferences. If 2 oral medications at maximum dosage do not adequately control thepatient’s blood glucose, leading to a A1C of l7.0%, the next step would be to start insulin therapy. The generalconsensus is that since it is the most effective therapy for achieving the recommended goal, the institution of insulinshould occur earlier in the therapeutic regimen than it does with many patients. If the idea is introduced early in theprocess and encouraged with the proper support, most patients will accept the use of insulin and be pleased by itseffectiveness in controlling their diabetes.

Bibliography 1. Nathan DM, et al: Management of hyperglycemia in type 2 diabetes: A consensus statement from the American Diabetes Association

and the European Association for the Study of Diabetes Mellitus. Diabetes Care 2006;29:1963–1972.

Item 54 Answer E

Although the UKPDS did not show a statisically significant difference in macrovascular outcomes with intensive vs.conventional therapy for diabetes, epidemiologic analysis of study data showed significantly lower CVD rates withlower achieved A1C levels, as described in options A–D. The reductions were incremental and the lower the A1Cwas maintained the lower the complication rate as described in options A–D. The rate for heart failure was actually only 16% reduction/1% drop in A1C as noted in Figure 4, much less than 42%.

51Educational (Learning) Critiques American Diabetes Association Self-Assessment Program

Figure 4.

Hazard ratios, with 95% confidence intervals as floating absolute risks, as estimate of association between category of updated mean A1C concentration andmyocardial infarction, stroke, microvascular end points, cataract extraction, lower extremity amputation or fatal peripheral vascular disease, and heart failure.Reference category (hazard ratio 1.0) is A1C l6% with log linear scales. P value reflects contribution of glycemia to multivariate model. Data adjusted for age atdiagnosis of diabetes, sex, ethnic group, smoking, presence of albuminuria, systolic blood pressure, high and low density lipoprotein cholesterol, and triglycerides.

Bibliography 1. Stratton IM, et al: Association of glycaemia with macrovascular and microvascular complications of type 2 diabetes (UKPDS 35):

prospective observational study. BMJ 2000;321:405–412.

Item 55 Answer B

All patients with diabetes should receive yearly influenza vaccinations. Patients with diabetes less than 65-years-oldshould receive a pneumococcal vaccine and it should be repeated once after the age of 65 if it has been L5 yearssince the last vaccination. The other vaccines mentioned are indicated only in special populations at increased risk.The rates of vaccination can be increased by using screening tools and standing order protocols developed by theCenters for Disease Control.

Fatal and non-fatal myocardial infarction

Haz

ard

ratio

14% decrease per1% reduction in A1C

10

1

0.5

P l0.0001

Fatal and non-fatal stroke

12% decrease per1% reduction in A1C

P = 0.035

Microvascular end points

Haz

ard

ratio

37% decrease per1% reduction in A1C

10

1

0.5

P l0.0001

Cataract extraction

19% decrease per1% reduction in A1C

P l0.0001

Amputation or death from peripheral vascular disease

Haz

ard

ratio

43% decrease per1% reduction in A1C

10

1

0.5 5 6 7 8 9 10 5 6 7 8 9 10

P l0.0001

Heart failure

Updated mean haemoglobin A1C concentration (%)

16% decrease per1% reduction in A1C

P = 0.021

American Diabetes Association Self-Assessment Program Educational (Learning) Critiques52

Bibliography 1. www.cdc.gov/nip/recs/adult-schedule.htm

2. www.immunize.org/catg.d/4036need.pdf (screening tool)

3. www.immunize.org/catg.d/p3075.pdf (standing orders for Pneumococcal vaccine)

Item 56 Answer C

The UK Prospective Diabetes Study (UKPDS) Group found that while type 2 diabetes is a progressive disease,on average, patients had already lost about half (50%) of their beta-cell function by the time they were diagnosed.This highlights the need to intervene with patients before frank diabetes is confirmed. Multiple interventionsincluding diet, exercise, weight loss and certain medications have been found to reduce the progression fromglucose intolerance to diabetes.

Bibliography 1. UK Prospective Diabetes Study Group. UK Prospective Diabetes Study 16. Overview of 6 years’ therapy of type 2 diabetes:

a progressive disease. Diabetes 1995;44:1249–1258.

Item 57 Answer D

Hypoglycemia generally occurs gradually and is preceded by warning signs, such as anxiety, diaphoresis,tachycardia and shakiness. The level of serum glucose that causes symptoms of hypoglycemia may vary frompatient to patient and even within the same patient at times. Impaired level of consciousness may occur withhypoglycemia but death is uncommon, unless warning signs are ignored for a prolonged period or a seriousmedication error is made. Death from low blood glucose levels alone rarely occurs unless it involves a dangeroussituation, e.g., while driving, climbing stairs, etc.

A recent study of adverse drug reactions that led to an emergency department visit showed that there are about700,000 such patients’ visits annually. One quarter of these visits were by patients over the age of 65 and almost3500 patients were admitted. The medications most commonly causing adverse events were insulins, oral hypo-glycemics. opioid-containing products, anticoagulants, amoxicillin-containing products, and antihistamines/coldproducts. One-third of adverse events in elderly patients were caused by warfarin, insulin, and digoxin. Over 85%of elderly patients who presented with an unintentional overdose were taking anti-hyperglycemic agents, warfarin,anti-convulsants, digoxin, theophylline, or lithium.

Bibliography 1. American Diabetes Association. Diabetes Care 2007;30(Suppl 1):S85.

2. Budnitz DS, et al: National surveillance of emergency department visits for outpatient adverse drug events. JAMA 2006;296:1858–66.

Item 58 Answer E

Insulin actually promotes the storage of triglycerides into adipose tissue, not the degradation of triglycerides intoFFAs. Therefore, if there is insulin resistance, the uptake of triglycerides is reduced and more FFAs are present inthe blood stream. All the other options (A, B, C, D) are consistent with the expected effects of insulin resistance.The normal suppression of gluconeogenesis by insulin in the liver is impaired. The normal uptake of glucose byskeletal muscle is reduced leading to elevated glucose levels. The normal glucose disposal, which includes glucoseoxidation and glycogen synthesis, by skeletal muscle, is impaired, raising the glucose levels further. Mildly elevatedlevels of FFAs do not impair β-cell function initially but actually increase insulin secretion to compensate for theearly phase of insulin resistance. As the FFA levels rise however, β-cell dysfunction and damage occurs and thecompensatory ability to increase insulin production is overwhelmed and relative insulin deficiency occurs.

Bibliography 1. Unger RH. Lipotoxic diseases. Ann Rev Med 2002;53:319–336.

53Educational (Learning) Critiques American Diabetes Association Self-Assessment Program

Item 59 Answer E

Environmental factors, such as diet and exercise, clearly impact the development of pancreatic β-cell dysfunction.Genetic factors, such as family history of diabetes or glucose intolerance increase the chances significantly of devel-oping the same conditions. Type 2 diabetes is 2–6 times more prevalent in African-Americans, Native Americans,Pima Indians, and Hispanic Americans in the US than in white Americans. In addition, 39% of patients with type 2diabetes have at least one parent with the disease. As discussed in the critique for Item 60, FFAs contribute topancreatic β-cell dysfunction by direct toxicity to the islet cells. Studies also suggest that glucose itself furtherincreases β-cell loss by several mechanisms including protein glycation and glucokinase down-regulation, leadingto reduced production of insulin and even β-cell death. There does not appear to be any direct insulin toxicitywhich contributes to pancreatic β-cell dysfunction.

Bibliography 1. Unger RH. Lipotoxic diseases. Ann Rev Med 2002;53:319–336

2. Klein BE, et al: Parental history of diabetes in a population-based study. Diabetes Care 1996;19:827.

3. Harris MI, et al: Prevalence of diabetes, impaired fasting glucose, and impaired glucose tolerance in U.S adults: the Third NationalHealth and Nutrition Examination Survey, 1988–1994. Diabetes Care 1998;21:518.

Item 60 Answer C

Option (A) This is diabetic dermopathy which often occurs as pigmented, pretibial, symmetric pink and brown-bronze atrophic shiny skin patches. In this picture we see the leg of a 59-year-old man withdiabetes. Although this condition is not specific to diabetes, it is probably the most common finding in the skin of diabetics. These pigmented pretibial patches may be a marker for underlying cutaneousmicroangiopathy.

Option (B) Acanthosis nigricans is a pigmenting disorder which causes velvety, light-brown-to-black markingsusually on the neck, under the arms, or in the groin. Acanthosis nigricans is most often associated withobesity and diabetes. It is thought to be caused by elevated insulin levels during the early phase ofinsulin resistance which activates insulin receptors in the skin, causing it to be more heavily pigmented.

Option (C) This is a case of scabies which is not specifically related to diabetes.

Option (D) This is a diabetic foot ulcer. These develop from a combination of neuropathy causing an underesti-mation of pressure on an area of the foot, and microangiopathy, poor blood flow and wound healing.

Bibliography 1. All of the images were borrowed with permission from:

Cohen BA, et al: DermAtlas. Johns Hopkins University, 2006. www.DermAtlas.org

Item 61 Answer E

Patients who develop type 2 diabetes mellitus have a progressive decline in β-cell function. The UKPDS showedthat at the time of diagnosis of type 2 diabetes mellitus one has lost p50% of β-cell function and continues to loseβ-cell function over time (Figure 5). The graph thus suggests that loss of β-cells may accrue for up to p10 yearsprior to the diagnosis of type 2 diabetes mellitus. This estimate is confirmed by retinopathy rates that exist at timeof diagnosis of type 2 diabetes mellitus compared to time known to develop retinopathy in type 1 diabetes wheretime of onset is more precisely defined (NHANES III). The question had been raised that if one screened for type2 diabetes mellitus in patients at risk, before they have presumably lost p50% of their β-cells, and found patientswith IGT, might one delay or prevent the onset of overt diabetes? Many studies, pictorialized in Figure 6, now haveshown reduced risk of developing type 2 diabetes mellitus with lifestyle changes, and with metformin in the DPP4study, with acarbose in the Stop-NIDDM trial and with rosiglitazone in the DREAM trial. However in the DREAMtrial as well, ramipril was not shown to delay or prevent type 2 diabetes mellitus. The ADA recently recommendeda combination of lifestyle changes and metformin for patients with IFG and IGT.

American Diabetes Association Self-Assessment Program Educational (Learning) Critiques54

Figure 5. Decline of ββ-cell function in the UKPDS illustrates progressive nature of diabetes.

Figure 6. Is it possible to delay the onset of type 2 diabetes mellitus?

Bibliography 1. Wajchenberg, BL. β-Cell Failure in Diabetes and Preservation by Clinical Treatment. Endocr Rev 2007;28(2):187–218.

2. Gillies. Pharmacological and lifestyle interventions to prevent or delay type 2 diabetes in people with IGT. BMJ 2007;334:299–302.

3. Jeon C. Physical Activity of Moderate Intensity and Risk of Type 2 Diabetes. Diabetes Care 2007;30:744–752.

4. Henness S. Pharmacologic interventions in the prevention of Type 2 Diabetes. Clin Op in Endocrinology, Diabetes And Obesity2007;14:166–169.

5. Holman RR. Diab Res Clin Pract 1998;40(suppl):S21–S25.

6. UKPDS. Diabetes 1995;44:1249–1258.

7. Nathan DM, et al: Diabetes Prevention Program (DPP). N Engl J Med 2002;346:393–403.

8. Chiasson JL, et al: Prevent Non-Insulin-Dependent Diabetes Mellitus (NIDDM). Lancet 2002;359:2072–77.

9. Buchanan T, et al: Troglitazone in the Prevention of Diabetes (TRIPOD). Diabetes 2002;51(9):27960–2803.

10. Torgerson JS, et al: Xenical in the Prevention of Diabetes in Obese Subjects (XENDOS). Diabetes Care 2004;27(1):155–61.

11. Gerstein H, et al: Diabetes Reduction Assessment with Ramipril & Rosiglitazone Medication (DREAM). Lancet 2006;368:1096–1105and NEJM 2006;355:1551–1562.

12. Nathan, et al: Impaired Fasting Glucose and Impaired Glucose Tolerance (PIOPOD), implications for care. Diabetes Care2007;30:753–759.

13. Harris MI, et al: Is the risk of diabetic retinopathy greater in non-Hispanic blacks and Mexican Americans than in non-Hispanic whiteswith type 2 diabetes? A U.S. population study. Diabetes Care 1998;21(8):1230–5.

Diabetes Prevention Clinical Trials

Dia

bete

s M

ellit

us R

educ

tion

(%)

70

60

50

40

30

20

10

0

58% 58%

31%25%

55% 55%

41%

62%

42%

Finnish

Da Qing – Diet + Exercise

DPP-Lifestyle

DPP-Metform

in

STOP-N

IDDM

TRIPOD

XENDOS

DREAM

PIOPOD

?100

80

60

40

20

0β-ce

ll fu

nctio

n (%

of n

orm

al b

y H

OM

A)

Years

Time of diagnosis

-10 -9 -8 -7 -6 -5 -4 -3 -2 -1 0 1 2 3 4 5 6

Pancreatic function = 50% of normal

55Educational (Learning) Critiques American Diabetes Association Self-Assessment Program

Item 62 Answer D

It is widely believed that a meal plan that focuses on the foods the patient is used to eating is most likely to engenderthe best compliance. This also fits with the concept that there is no specific prescription for proportion of carbohy-drate, fat, and protein combinations that are specifically proven to result in better care of the patient with diabetes.This also implies that if one tries to change the foods the patient is used to eating, then, by definition they will, mostlikely, not be able to follow the diet even in the short-term, or especially the long-term. So nutrition counseling thatis individualized holds the best chance for success in fulfilling the Nutrition recommendations of the ADA.

Extracts from ADA recommendation:

“The Dietary Reference Intakes (DRI) report of the Institute of Medicine recommends that, to meet thebody’s daily nutritional needs while minimizing risk for chronic diseases, healthy adults should consume45–65% of total energy from carbohydrate, 20–35% from fat, and 10–35% from protein. It must be clearlyrecognized that regardless of the macronutrient mix, total caloric intake must be appropriate to weightmanagement goals.”… Sugar alcohols and non-nutritive sweeteners are safe when consumed within the dailyintake levels established by the FDA… The FDA has approved five non-nutritive sweeteners for use in theU.S. These are acesul fame potassium, aspartame, neotame, saccharin, and sucralose. Before being allowedon the market, all underwent rigorous scrutiny and were shown to be safe when consumed by the public,including people with diabetes and women during pregnancy. Substantial evidence from clinical studiesdemonstrates that dietary sucrose does not increase glycemia more than isocaloric amounts of starch.[Patients with] “type 2 diabetes should be encouraged to achieve the US Department of Agriculture (USDA)recommendation for dietary fiber (14 g fiber/1000 kcal)” …A dietary pattern that includes carbohydratefrom fruits, vegetables, whole grains, legumes, and low-fat milk is encouraged for good health.

Bibliography 1. Institute of Medicine: Dietary Reference. Intakes: energy, carbohydrate, fiber, fat, fatty acids, cholesterol, protein, and amino acids.

National Academies Press, Washington, DC. 2002.

2. Nutrition Recommendations and Interventions for Diabetes. Diabetes Care 2007;30(Suppl 1):S48–S65.

3. Franz MJ, et al: Evidence-based nutrition principles and recommendations for the treatment and prevention of diabetes and related complications. Diabetes Care 2002;25:148–198.

Item 63 Answer A

A characteristic syndrome has been described in African-Americans, often seen in patients with new-onset diabeteswho present with DKA. After initial usual aggressive care of their DKA and its precipitant, they are able to beweaned from insulin and act as typical type 2 patients. It is felt to be a genetically specific atypical diabetes, mostlikely related to the ability of the β-cell to recover insulin secretion function, possibly initially suppressed by anundue glycotoxicity or lipotoxicity. African-Americans do not have any particular extra responsiveness to TZDs.They have an increased incidence of family history of diabetes, as do other patients with type 2 diabetes. There isno association with sickle cell trait and the expected low proportions of type 2 patients who have anti-islet cellantibodies. Option (A) is the correct answer, the ability to stop insulin therapy soon after presentation with DKA.

Bibliography 1. Banerji MA, et al: GAD antibody negative NIDDM in adult black subjects with diabetic ketoacidosis and increased frequency of human

leukocyte antigen DR3 and DR4. Flatbush diabetes. Diabetes 1994;43:741–745.

2. Pinero-Pilona A, et al: Idiopathic type 1 diabetes. J Diabetes Complications 2001;15:328–335.

3. Maldonado M, et al: Ketosis-prone diabetes: dissection of a heterogeneous syndrome using an immunogenetic and ß-cell functionalclassification, prospective analysis, and clinical outcomes. J Clin Endocrinol Metab 2003;88:5090–5098.

4. Kitabchi AE. Ketosis-prone diabetes: a new subgroup of patients with atypical type 1 and type 2 diabetes? J Clin Endocrinol Metab2003;88:5087–5089.

5. Wajchenberg BL. β-Cell Failure in Diabetes and Preservation by Clinical Treatment. Endocr Rev 2007;28(2):187–218.

American Diabetes Association Self-Assessment Program Educational (Learning) Critiques56

Item 64 Answer C

The ADA believes a A1C less than 7% is a reasonable goal for glycemic control. A preprandial capillary plasmaglucose level should be 90–130 mg/dL and peak post-prandial capillary plasma glucose l180 mg/dL. The ADAalso recommends that A1C goals should be as low as possible, even l6%, but, most importantly, as low as possiblewithout undue hypoglycemia. So in this patient option (C) would be the correct answer, a fasting glucose levell140 mg/dL would not be considered low enough.

The discussion in consensus conferences had to do with the accepted data that damage to tissues starts with post-prandial glucose levels L140 mg/dL (DECODE, Honolulu Heart Study), and with DCCT, UKPDS, andEDIC trials we know that decrease in A1C reduces risks of complications, but that risk of hypoglycemia, especiallywith recommended therapies of secretogogues and insulin, engender significant risks of hypoglycemia. So at asocietal level, one would not want to cause undue hypoglycemia in a whole population because of goals that mightbe too “tight.”

Table 3. Intensive therapy for diabetes: Reduction in incidence of complications.

DCCT Research Group. N Engl J Med 1993;329:977–986. Ohkubo Y, et al: Diabetes Res Clin Pract 1995;28:103–117.UKPDS 33: Lancet 1998;352:837–853.Adapted from D. Kendall, International Diabetes Center

Type 1 Type 2 Type 2DCCT Kumamoto UKPDS

A1C 9 R 7% 9 R 7% 8 R 7%

Retinopathy 63% 69% 17–21%

Nephropathy 54% 70% 24–33%

Neuropathy 60% – –

Cardiovascular disease 41% – 16%

57Educational (Learning) Critiques American Diabetes Association Self-Assessment Program

Figure 7. DDCT/EDIC Trial: Reduction in CV outcomes by glycemic control with insulin therapy.

Bibliography 1. Hanefeld M, et al: Diabetologia 1996;39:1577–1583.

2. Decode Study Group. Lancet 1999;354:617–21.

3. Donahue RP, et al: Diabetes 1987;36:689–692.

4. DCCT Research Group. The effect of intensive diabetes treatment on the development and progression of long-term complications in insulin-dependent diabetes mellitus: Diabetes Control and Complications Trial (DCCT). N Eng J Med 1993;329:977–86.

5. Harris MI, et al: Is the risk of diabetic retinopathy greater in non-Hispanic blacks and Mexican Americans than in non-Hispanic whiteswith type 2 diabetes? A U.S. population study. Diabetes Care 1998;21(8):1230–5.

6. American Diabetes Association. Clinical Practice Recommendation. 1999.

7. DCCT Group. Diabetes Care 1995;44:968–983.

8. DCCT/EDIC Research Group. The effect of intensive diabetes treatment of cardiovascular disease in type 1 diabetes: The Diabetes Control and Complications Trial/Epidemiology of Diabetes Interventions and Complications (EDIC) Study.N Eng J Med 2005;353:2643–2653.

9. United Kingdom Prospective Diabetes Study (UKPDS) Group. Intensive blood glucose control with sulphonylureas or insulin compared with conventional treatment and risk of complications in patients with type 2 diabetes (UKPDS 33).Lancet 1998;352:837–53.

Cum

ulat

ive

Inci

denc

e42% Risk ReductionP + 0.017

Any Cardiovascular Event

Non-Fatal MI, Non-Fatal Stroke or Cardiovascular Death

58% Risk ReductionP = 0.18

DCCT

A1C

9.2

% v

s 7.

1% fo

rM

ean

6.5

year

s; E

DICT

avg

A1C

8.0

Years Since Entry

0.12

0.10

0.08

0.06

0.04

0.02

0.00 0 1 2 3 4 5 6 7 8 9 10 11 12 13 14 15 16 17 18 19 20 21

Number at Risk Intensive: 705 683 629 113Conventional: 714 688 618 92

Years Since Entry

0.12

0.10

0.08

0.06

0.04

0.02

0.00 0 1 2 3 4 5 6 7 8 9 10 11 12 13 14 15 16 17 18 19 20 21

Number at Risk Intensive: 705 686 640 113Conventional: 721 694 637 96

Conventional

Conventional

Intensive

Intensive

American Diabetes Association Self-Assessment Program Educational (Learning) Critiques58

Item 65 Answer E

The current ADA guidelines suggest metformin should be started at the onset of therapy for all the reasonsmentioned in this item.

K It improves insulin resistance in liver, decreasing gluconeogenesis and glycogenolysis. It has a high initialresponse rate, drops A1C p 2%. There is no initial weight gain and can have a modest initial weight loss.

K It is not associated with hypoglycemia when used alone or with TZD, exenatide, or DPP-4 Inhibitors.K It has potential to delay or prevent type 2 diabetes mellitus and progression (DPP-2 trial), but its secondary

failure is only modestly better than sulfonylureas (ADOPT trial).K It decreases MIs 39% in the retrospective analysis in the UKPDS obese subgroup, it decreases advanced

glycosylated end-products (AGEs), and improves endothelial dysfunction.K Disadvantages – One must watch for:

k GI side-effects on initiation, though titration up to maximal effective dose of 2 g/day over a month decreases the risk.

k Delay initiation until Cr levels are proven stable using radiologic iodinated contrast media studies. This willreduce risk of lactic acidosis. Moreover, don’t use if have significantly impaired hepatic function or cardio-vascular compromise (CHF) or Cr L1.4 female, L1.5 male.

k Blood levels of metformin increase in patients with Cr clearance l70 (age L70). Patients with Cr Clearancel40 is where most cases of lactic acidosis cases have occurred. Thus, clinical judgement must be used inpatients age 70–80 (the age the product insert recommends stopping or not using metformin).

Figure 8. Advanced glycation end products.

Figure 9. Guanidino compounds that bind dicarbonyls.

NH2 NH2C NH

NH

NH2 C N

CH3

CH3

C NH

NH NH

Aminoguanidine Metformin(Dimethylbiguanide)

Aminoguanidine (AMG)

D-Glucos e +Protei n (”R” ) Schif f Ba se

AmadoriProduct AP- ene-dione

A.G .E.Cro sslinked Pr oteins

HOHO OH

OHOH

CHO HOHO OH

OHOH

+NHR HO OH

OHOH

NHROOO

HO O

OH

NHR HN[Pro tei n]

[Pro tei n]O

XOH

+RNH2

59Educational (Learning) Critiques American Diabetes Association Self-Assessment Program

Figure 10. Sources of methylglyoxal.

Figure 11. Media levels of methylglyoxal with or without metformin.

Bibliography 1. Nathan DM, et al: Management of hyperglycemia in type 2 diabetes: A consensus algorithm for the initiation and adjustment of therapy.

A consensus statement from the American Diabetes Association and the European Association for the Study of Diabetes. Diabetes Care2006;29:1963–1972.

2. Bailey CJ, et al: Metformin. N Engl J Med 1998;334:574–583.

3. DeFronzo R, et al: Metformin Study Group. Efficacy of metformin in patients with non-insulin-dependent diabetes mellitus.N Engl J Med 1995;333:541.

4. Diabetes Prevention Program Research Group: Reduction in the incidence of type 2 diabetes with lifestyle intervention or metformin.N Engl J Med 2002;346:393–403.

5. Kahn SE, et al: ADOPT trial. N Engl J Med 2006;355:2427–43.

6. UKPDS Group. Effect of intensive blood-glucose control with metformin on complications in overweight patients with type 2 diabetes(UKPDS 34). Lancet 1998;352:854–865.

7. Rahbar S, et al: Evidence that pioglitazone, metformin and pentoxifylline are inhibitors of glycation. Clin Chem Acta2000;301(1–2):65–77.

8. Ruggiero-Lopez D, et al: Reaction of metformin with dicarbonyl compounds. Possible implication in the inhibition of advancedglycation end product formation. Biochem Pharmacol 1999;58(11):1765–73.

9. Vitale C, et al: Metformin improves endothelial function in patients with metabolic syndrome. J Intern Med 2005;258(3):250–6.

Treatment Group+ Metformin - Metformin

Med

ia M

ethy

lgly

oxal

(pm

ols/

gm R

BCs

)

4000

3500

3000

2500

2000

1500

1000

500

0

P = 0.03

Fructoselysine

MethylglyoxalTriose Phosphates

Dihydroxyacetone PO4

Glyceraldehyde-3-PO4

Acetol

Ene-diol

GAPDH

Mono-oxygenase

Mono-oxygenaseP = 450

L-ThreonineDehydrogenase

SSAO

Acetone

Serine

Aminoacetone

Glucose

L-LactatePyruvate

Ketone BodiesAcetoacetate

β-Hydroxybutyrate

American Diabetes Association Self-Assessment Program Educational (Learning) Critiques60

Item 66 Answer E

Metformin reduces insulin levels and therefore it would reduce C-peptide levels. In fact, metformin does reducehepatic glucose output, hepatic gluconeogenesis, insulin levels, and fasting blood glucose levels.

Bibliography 1. Macfarlane DP, et al: Oral antidiabetic agents as cardiovascular drugs. Diabetes Obes Metab 2007;9(1):23–30.

2. Chan. Role of metformin in the initiation of pharmacotherapy for type 2 diabetes: an Asian-Pacific perspective. Diabetes Res Clin Prac2007;75(3)255–66.

3. Bailey CJ, et al: Metformin. N Eng J Med 1996;334:574.

4. Schimmack G, et al: AMP-activated protein kinase: Role in metabolism and therapeutic implications. Diabetes Obes Metab2006;8(6):591–602.

5. Guigas B, et al: 5-Aminoimidazole-4-carboxamide-1-beta-D-ribofuranoside and metformin inhibit hepatic glucose phosphorylation by an AMP-activated protein kinase-independent effect on glucokinase translocation. Diabetes 2006;55(4):865–74.

6. Natali A, et al: Effects of metformin and thiazolidinediones on suppression of hepatic glucose production and stimulation of glucose uptake in type 2 diabetes: a systematic review. Diabetologia 2006;49(3):434–41.

Item 67 Answer B

A general principle of combination therapy for treatment of hyperglycemia in type 2 diabetes without using insulin is that each agent will reduce A1C an additional 1%. There is a logic, though no published studies, to use a combination of TZD, metformin and an incretin mimetic to control blood glucose levels avoiding an undue risk of hypoglycemia. One achieves benefits of glycemic control, potential CV benefit, potential β-cell preservation,as well as decreased endothelial dysfunction and inflammation. Moreover, if exenatide is used, it is likely to achievesignificant weight loss. There is no added benefit in using two TZDs (rosiglitazone and pioglitazone).

Bibliography 1. Davidson MB. Triple therapy: definitions, application, and treating to target. Diabetes Care 2004;27(7):1834–5.

2. Hamid Z, et al: Triple therapy in type 2 diabetes: insulin glargine or rosiglitazone added to combination therapy of sulfonylurea plusmetformin in insulin-naive patients: response to Rosenstock, et al. Diabetes Care 2006;29(10):2331.

3. Dorkhan M, et al: Glycaemic and nonglycaemic effects of pioglitazone in triple oral therapy of patients with type 2 diabetes.J Intern Med 2006;260(2):125–33.

4. Tran MT, et al: Comparison of the glycemic effects of rosiglitazone and pioglitazone in triple oral therapy in type 2 diabetes.Diabetes Care 2006;29(6):1395–6.

5. Rosenstock J, et al: Triple therapy in type 2 diabetes: insulin glargine or rosiglitazone added to combination therapy of sulfonylurea plus metformin in insulin-naive patients. Diabetes Care 2006;29(3):554–9.

6. Roberts VL, et al: Triple therapy with glimepiride in patients with type 2 diabetes mellitus inadequately controlled by metformin and a thiazolidinedione: results of a 30-week, randomized, double-blind, placebo-controlled, parallel-group study. Clin Ther2005;27(10):1535–47.

Item 68 Answer C

Increased quantity of visceral fat is felt to potentiate, by several different mechanisms, muscle and liver insulinresistance, reduce insulin secretion and is also is directly related to increasing vascular endothelial dysfunction.

In the past, it was felt that insulin resistance was an issue purely of muscle with reduced glucose uptake, and liver withincreased glucose production; but several years ago, it was learned it is about fat, not fat in the buttocks and thethighs, but visceral obesity, the apple physiognomy, the beer belly, the increased visceral fat that accounts for increasedinsulin resistance, and is also associated with diabetes and its complications. When the individual fat cell increases insize and the total visceral fat quantity increases in volume, they become endocrine organs. They secrete cytokines andtoo much of some hormones like angiotensinogen, and too little of others like adiponectin. This affects muscle-reducing glucose uptake, liver-increasing glucose production, and actually affects the pancreas in reducing furtherβ-cell secretion. Moreover, the body attempts to reduce that volume and quantity of visceral fat by shifting free fattyacids to the periphery; and the free fatty acids also go to the muscle, reduced glucose uptake, go to the liver increasing

61Educational (Learning) Critiques American Diabetes Association Self-Assessment Program

glucose production, also, in fact, reduce β-cell function even more and starts the diabetic state. Moreover, thecombination of FFAs, cytokines, and abnormal hormone secretions are directly related to endothelial dysfunctionand inflammatory changes in the arterial epithelium; and this is exacerbated by CRP and PAI-1 production by theinsulin-resistant liver, which all go to the lining of the arterioles and start the atherosclerotic process. Thus, there isa clear relationship between obesity, insulin resistance, diabetes and atherosclerotic disease.

Bibliography 1. Rader DJ. Effect of insulin resistance, dyslipidemia, and intra-abdominal adiposity on the development of cardiovascular disease

and diabetes mellitus. Am J Med 2007;120(3 Suppl 1).

2. Bergman RN, et al: Abdominal obesity: role in the pathophysiology of metabolic disease and cardiovascular risk.Am J Med 2007;120(2 Suppl 1):S3–8.

Item 69 Answer B

Post-prandial hyperglycemia is the earliest detectable glycemic abnormality in the majority of patients withdiabetes; 80% of them present in this way. It contributes to the A1C level of 6.5% to a greater extent than doesfasting blood glucose. It has been associated with an increased risk of microvascular complications in theDECODE study and Honolulu heart study. It has been associated with an increased risk of macrovascular compli-cations in the NHANES study as well as an interpretation of the DCCT results. Even with A1C at goal, 75% ofpost-prandial glucose levels will be elevated and, indeed, with A1C of 6.5% roughly 75% of this value is a result of post-prandial hyperglycemia. There is one study that suggests that by controlling post-prandial glucose levelsadverse outcomes may be reduced i.e., complications of pregnancy in women with diabetes (decrease large for-dates babies by 2/3, decrease C-sections by 2/3, and almost eliminate neo-natal hypoglycemia). However, otherthan that study, there is no data that proves that reducing post-prandial hypoglycemia will, in fact, reduce othercomplications or cardiovascular outcomes. So, the correct answer is option (B), that no convincing data exists thattreating post-prandial glucose elevations will reduce cardiovascular outcomes or other complications of diabetes.

Bibliography 1. Yamagishi I, et al: Role of post-prandial hyperglycaemia in cardiovascular disease in diabetes. Int J Clin Prac 2007;61(1)83–87.

2. Rendell MS, et al: Targeting post-prandial hyperglycemia. Metabolism 2006;55(9):1263–81.

3. Leiter LA, et al: International Prandial Glucose Regulation (PGR) Study Group. Post-prandial glucose regulation: New data and newimplications. Clin Ther 2005;27(Suppl 2):S42–56.

4. Robertson C. Physiologic insulin replacement in type 2 diabetes: optimizing post-prandial glucose control. Diabetes Edu2006;32(3):423–32.

5. Brindisi MC, et al: Post-prandial hyperglycaemia: to treat or not to treat? Diabetes Metab 2006;32(2):105.

6. Gerich JE. Post-prandial hyperglycemia and cardiovascular disease. Endocr Pract 2006;12(Suppl 1):47–51.

7. Mecacci F, et al: Maternal metabolic control and perinatal outcome in women with gestational diabetes treated with regular or lisproinsulin: comparison with non-diabetic pregnant women. Eur J Obstet Gynecol Reprod Bio 2003;111:19–24.

8. de Veciana M, et al: Post-prandial versus pre-prandial blood glucose monitoring in women with gestational diabetes mellitus requiringinsulin therapy. N Engl J Med 1995;333:1237–1241.

Item 70 Answer C

Fast analog insulins were developed to mimic normal physiology of prandial insulin, release and duration. Studieshave shown that they can reduce hypoglycemia overnight or during the day. They can decrease post-prandialhypoglycemia by its rapid onset and loss of effect over several hours. As part of a basal/bolus protocol of insulintherapy, it allows the patient to eat when they wish, skip or delay meals, eat varying quantity of meals, do shift work,travel across time zones, wake up different times on weekends and weekdays and in that sense instead of the diseasetherapy controlling ones lifestyle, i.e., must eat at the same time everyday, the patient can have freedom of lifestyle,have control over their disease. Though we would have liked to have seen studies confirming reduction consistentlyin A1C, this has not occurred as yet in the published studies.

American Diabetes Association Self-Assessment Program Educational (Learning) Critiques62

Bibliography 1. Robertson C. Physiologic insulin replacement in type 2 diabetes: optimizing post-prandial glucose control. Diabetes Edu

2006;32(3):423–32.

2. Mudaliar SR, et al: Insulin aspart (B28 asp-insulin): a fast-acting analog of human insulin: absorption kinetics and action profilecompared with regular human insulin in healthy nondiabetic subjects. Diabetes Care1999;22(9):1501–6.

3. Heller SR, et al: Effect of the fast-acting insulin analog lispro on the risk of nocturnal hypoglycemia during intensified insulin therapy.U.K. Lispro Study Group. Diabetes Care 1999;22(10):1607–11.

4. Bruttomesso D, et al: Restoration of early rise in plasma insulin levels improves the glucose tolerance of type 2 diabetic patients.Diabetes 1999;48(1):99–105.

5. Dailey G, et al: Insulin glulisine provides improved glycemic control in patients with type 2 diabetes. Diabetes Care 2004 ;27(10):2363–8.

Item 71 Answer A

Multiple metabolic mechanisms have now been implicated in the etiology of complications due to hyperglycemia.Strong data exists in the literature for the role of increased flux in the aldose reductase pathway, production of advanced glycosylation end products, increased protein kinase C pathway flux, increase flux through thehexosamine pathway and hyperglycemia increasing oxidative stress in mitochondria. As a result they have multipleaffects in cells and in vascular endothelium and have been strongly associated with the production of eye, kidney,and nerve disease.

There is also a pentose-phosphate shunt pathway that takes fructose 6-phosphate and creates, via transketolase,pentose-phosphate. One can give a medicine, benfotiamine, a fat-soluble thiamine relative, that will increasepentose-phosphate shunt flux and in doing so will reduce flux in several of the above pathways. Thus, the correctanswer is increasing pentose-phosphate shunt flux – it can have the potential to reduce complications of diabetes,not increase them.

Figure 12.

Bibliography 1. Brownlee M. The pathobiology of diabetic complications: A unifying mechanism. Diabetes 2005;54.

2. Brownlee M. Biochemistry and molecular cell biology of diabetic complications. Nature 2001;414:813–820.

Pentose-5-phosphates

+Erythrose-4-phosphate

TK

Thiamine

q Glucose

q Glucose-6-P

q Fructose-6-P

q Glyceraldehyde-3-P

q Sorbitol

q Glucosamine-6-P q UDP-GlcNAc

q Fructose

1, 3-Diphosphoglycerate

NAD+

NADH

NADPH

GFAT

NADHNADP+

Gln Glu

NAD+

Q GAPDH q O2

Polyol pathway

Hexosamine pathway

q DHAP q α-Glycerol-P q DAG q PKCNADH NAD+

Protein kinase C pathway

q Methylglyoxal q AGEs

AGE pathway

63Educational (Learning) Critiques American Diabetes Association Self-Assessment Program

Item 72 Answer D

Improved outcomes in patients with diabetes are seen when the blood pressure is under 130/80 mm Hg.This was seen in the UK PDS and HOT trials. The recommendation of an ACE-inhibitor (ACE-I) or an ARB isfelt to be important for their additional benefit in reducing microalbuminuria if it exists. Patients with diabetes mayrequire combination therapy with an ACE-I or an ARB. Although beta-blockers are often needed especially in thepost MI state, it may increase the risk of hypoglycemic unawareness and, in the patient with peripheral vasculardisease, by leaving leaving unopposed alpha sympathetic activity, can result in new symptomatic claudication; thusthe physician must alert the patient in this regard. HCTZ can, in fact, increase blood glucose level but this is felt tobe a relative contraindication and not an absolute contraindication as option (D) states. Thus, the correct answer isoption (D). Thus clinically, in a patient with newly-diagnosed diabetes, one might avoid HCTZ as it may make thedifference between lifestyle therapy and need for monotherapy for glycemic control or between mono and combi-nation therapy for glycemic control. However, in patients already on combination or insulin therapy, its effect onglycemic control is felt to be minimal and can easily be adjusted.

Bibliography 1. Hansson L, et al: HOT trial. Lancet 1998;351:1755–1762.

2. UKPDS Group. BMJ 1998;317:703–713.

3. Goodman, et al: Textbook of pharmacology [re: sulfonylurea and beta-blocker discussion].

4. Russo D, et al: Am J Kidney Dis 1999;33:851–856.

5. Parving HH, et al: N Engl J Med 2001;345:870–888.

6. Viberti GC, et al: Circulation 2002;106:672–678.

7. Lewis EJ, et al: N Engl J Med 2001;345:851–860.

8. Brenner BM, et al: N Engl J Med 2001;345:861–869.

Items 73–75 Answers 73 (B); 74 (A); 75 (C)

Morbidly obese patients would certainly benefit from weight loss. Weight loss has been shown to decrease bloodglucose levels, decrease insulin resistance, and decrease correlates of insulin resistance including endothelialdysfunction, inflammatory markers, dysfibrinolysis, insulin levels, blood pressure, and dyslipidemia. According to 2-year published data (Figure 13), exenatide is the only drug for diabetes that seems to have a persistent effect on reduction in weight. In 30 weeks of treatment, 25% of patients lost 5–10 lbs, 25% lost 10–20 lbs and 25% lostL20 lbs. Weight loss seems to continue for at least 2 years.

In general, each additional agent in combination therapy of patients with type 2 diabetes drops the A1C 1% point.Thus, even if 2 additional agents were added in the patient with a A1C of 9.6%, already on metformin, it would notbring the A1C to even ADA goals of control. Thus, insulin, as the most “effective agent” to reduce blood glucose,is the best option in this example.

A long-distance truck driver with diabetes on insulin therapy can no longer hold a commercial driver’s license.In addition, this truck driver is fearful of injections. Thus, a DPP-4 inhibitor, such as sitagliptin, would be the bestchoice with this patient.

American Diabetes Association Self-Assessment Program Educational (Learning) Critiques64

Figure 13. Change in body weight following 82 weeks of exenatide treatment.

BL = baselineBlonde L, et al: Diabetes Obes Metab 2006;8.436–447.

Bibliography 1. Buse JB, et al: Metabolic effects of two years of exenatide treatment on diabetes, obesity, and hepatic biomarkers in patients with type 2

diabetes: An interim analysis of data from the open-label, uncontrolled extension of three double-blind, placebo-controlled trials.Clin Ther 2007;29(1):139–53.

2. Ratner RE, et al: Long-term effects of exenatide therapy over 82 weeks on glycaemic control and weight in overweight metformin-treatedpatients with type 2 diabetes mellitus. Diabetes Obes Metab 2006;8(4)419–428.

3. Phelan S, et al: Impact of weight loss on the metabolic syndrome. Int J Obes 2007 Mar 13.

4. Kakafika AI, et al: The role of endocannabinoid system blockade in the treatment of the metabolic syndrome. J Clin Pharmacol2007;47(5):642–52.

5. DeFronzo RA, et al: Diabetes Care 2005;28:1092–1100.

6. Blonde L, et al: Diabetes Obes Metab 2006;8:436–447.

7. Deeks ED, et al: Pioglitazone/metformin.Drugs 2006;66(14):1863–77; discussion 1878–80.

8. Wellington K. Rosiglitazone/Metformin. Drugs 2005;65(11):1581-92; discussion 1593–4.

9. Buse JB, et al: Metabolic effects of two years of exenatide treatment on diabetes, obesity, and hepatic biomarkers in patients with type 2diabetes: An interim analysis of data from the open-label, uncontrolled extension of three double-blind, placebo-controlled trials.Clin Ther 2007;29(1):139–53.

10. Nauck MA, et al: Sitagliptin Study 024 Group. Efficacy and safety of the dipeptidyl peptidase-4 inhibitor, sitagliptin, compared with the sulfonylurea, glipizide, in patients with type 2 diabetes inadequately controlled on metformin alone: a randomized, double-blind,non-inferiority trial. Diabetes Obes Metab 2007;9(2):194–205.

11. Blickle JF. Meglitinide analogues: a review of clinical data focused on recent trials. Diabetes Metab 2006;32(2):113–20.

12. Riddle MC, et al: Insulin Glargine 4002 Study Investigators. The treat-to-target trial: randomized addition of glargine or human NPH insulin to oral therapy of type 2 diabetic patients. Diabetes Care 2003;26(11):3080–6.

13. Raskin P, et al: INITIATE Study Group. Initiating insulin therapy in type 2 Diabetes: a comparison of biphasic and basal insulinanalogs. Diabetes Care 2005 Feb;28(2):260–5.

<2525 to<30

30 to<35

35 to<40 >40

Baseline BMI (kg/m2)

n=11

n=91

n=97 n=64

n=51

Open-label ext ensions(10 µg bid EXE)

Chan

ge in

Bod

y W

eigh

t (kg

)

Chan

ge in

Bod

y W

eigh

t (kg

)

Placebo -controlledtrials

82-w eek complet er (n=314)Int ent-to-treat (n=551)

-3.5 ± 0.2 kg

-4. 4 ± 0.3 kg

0 10 20 4030 706050 80 90

0

-1

-2

-3

-4

-5

0

-2

-4

-6

-8

-10

Treatment (weeks )

65American Diabetes Association Self-Assessment Program

ADA-SAP 1 PDF Version – Multiple Choice Questions – Section A

76

77

78

79

80

81

82

83

84

51

52

53

54

55

56

57

58

26

27

28

29

30

31

32

33

34

1

2

3

4

5

6

7

8

9

1 26 51 76

2 27 52 77

3 28 53 78

4 29 54 79

5 30 55 80

6 31 56 81

7 32 57 82

8 33 58 83

9 34 59 84

10 35 60 85

11 36 61 86

12 37 62 87

13 38 63 88

14 39 64 89

15 40 65 90

16 41 66 91

17 42 67 92

18 43 68 93

19 44 69 94

20 45 70 95

21 46 71 96

22 47 72 97

23 48 73 98

24 49 74 99

25 50 75 100

65

© 2007 Professional Evaluation, Inc. All rights reserved. Please see page 66 for Program Evaluation Form

66 © 2007 Professional Evaluation, Inc.66

ADA-SAP 1 Program FeedbackThank you for participating in the ADA-SAP Module 1: Basic Principles of Diagnosis and Management of Type 2Diabetes. If you would like to provide any feedback on this or other Self-Assessment Programs from PEI, pleaseemail your comments to [email protected]. PEI looks forward to your continued participation in new programsof continuing education as they become available.